PPS COMPILED SAMPLEX [PART 3 OF 5] - 652 items total with Rationale Flashcards

1
Q
"A case of 6yo cough. After 3-4 days became productive. On Pe, noted with coarse crackles, occasional wheeze rr 38. Labs: wbc 20000 neutrophilic predominance
A. Acute bronchitis
B. Nonsevere pneumonia
C. Acute brochiolitis
D. Bacterial tracheitis"
A

B

“Nelson 21st p3335
GBS may follow administration of vaccines against rabies, influenza, and conjungated meningococcal vaccine.

PAPP 2021 PCAP CPG p15-16
Criteria for severe pneumonia (requiring admission)

  1. Respiratory signs - cyanosis/hypoxemia, head bobbing, retractions, apena, grunting
  2. CNS signs - lethargic/stuporous/comatose/GCS <13, seizures
  3. Circulatory signs - capillary refill >3s or shock, pallor
  4. General considerations - moderate to severe malnutrition, refusal or inability to take food/drink per orem, some to severe dehydration, age <6 months
  5. Ancillary parameters - CXR or UTZ findings of consolidation, multifocal disease, moderate to large effusion, abscess, air leak; sustained O2 sat at room air <=93% “
How well did you know this?
1
Not at all
2
3
4
5
Perfectly
2
Q
"Patient with painless purging of rice-water stools with fishy odor? Drug of choice?
A. Doxycycline
B. Pen G
C. Ciprofloxaxin
D. TMP-SMX"
A

A

“Nelson 21st p1516
Following an incubation period of 1-3 days, acute watery diarrhea and vomiting ensue. Diarrhea can progress to painless puring of profuse rice-water stools (suspended flecks of mucus) with a fishy smell, which is the hallmark of the disease.

Cholera gravis, the most severe form of the disease, results when puring rates of 500-1000 ml/hr occur.

Antibiotics should only be given in patients with moderately severe to to severe dehydration. Antibiotics shorten the duration of illness, decrease fecal excretion of vibrios, decrease the volume of diarrhea, and reduce the fluid requirement during rehydration.

Single dose antibiotics increase compliance; doxycycline, ciprofloxacin, and azithromycin are effective against cholera. “

How well did you know this?
1
Not at all
2
3
4
5
Perfectly
3
Q
"What is a major risk factor for poor prognosis in meningococcemia?
A. Hypertension
B. Petichiae more than 24 hrs
C. Seizure
D. None of the above"
A

C

“Nelson 21st
Poor prognostic factors on presentation include hypothermia or extreme hyperpyrexia, hypotension or shock, purpura fulminans, seizures, leukopenia, thrombocytopenia (including DIC), acidosis, and high circulating levels of endotoxin and TNF-α. The presence of petechiae for <12 hr before admission, absence of meningitis, and low or normal ESR indicate rapid, fulminant progression and poorer prognosis”

How well did you know this?
1
Not at all
2
3
4
5
Perfectly
4
Q
"What is the least helpful in the diagnosis of meningo infection?
A. Isolation from petichiae and purpura
B. Isolation from nasopharynx
C. Titers from csf studies
D. Blood culture"
A

B

“Nelson 21st p1472
The initial diiagnosis of meningococcal disease should be made on clinical assessment to avoid delay in implementation of appropriate therapy.

A confirmed diagnosis of meningococcal disease is established by isolation of N. meningitides from a normally sterile body fluid such as blood, CSF, or synovial fluid. Meningococci may be identified in a gram stain preparation and/or culture of petechial or purpuric skin lesions, although this procedure is rarely undertaken, and occasionally are seen on Gram stain of the buffy coat layer of a centrifuged blood sample.

Isolation of the organism from the nasopharynx is not diagnostic of invasive disease because the organism is a common commensal. “

How well did you know this?
1
Not at all
2
3
4
5
Perfectly
5
Q
"Case of mother with chlamydia. What will you do with newborn?
A. Oral erythromycin for 14 days
B. Observe until symptoms arise 
C. Oral clarithromycin for 14 days 
D. Amoxicillin for 7 days"
A

A

“Nelson 21st p1617-1618
The most effective method of controlling perinatal chlamydial infection is screening and treatment of pregnant women.

The recommended treatment regimens for C. trachomatis conjunctivitis or pneumonia in infants are:

  1. Erythromycin (50mg/kg/day QID PO x 14 days)
  2. Azithromycin (20mg/kg/day OD PO x 3 days) “
How well did you know this?
1
Not at all
2
3
4
5
Perfectly
6
Q
"Case of SSPE. Most important part of history?
A. History of rubeola
B. History of trauma
C. Depression
D. None of the above"
A

A

“Nelson 21st p1673-1674
Subacute sclerosing panencephalitis (SSPE) is a chronic complication of measles with a delayed onset and an outcome that is nearly always fatal.

The diagnosis of SSPE can be eastablished through documentation of a complatible clinical course and at least one of the following supporting findings:

  1. Measles antibody detected in CSF
  2. Characteristic EEG findings
  3. Typical histologic findings in and/or isolation of virus or viral antigen from brain tissue obtained by biopsy or postmortem examination”
How well did you know this?
1
Not at all
2
3
4
5
Perfectly
7
Q
"Case of strider with hoarseness and dysphagia. Most important part of diagnosis?
A. History and PE
B. Chest xray
C. Blood CS
D. CBC"
A

A

“Nelson 21st p2203
Most patients have an URTI with some combination of rhinorrhea, pharyngitis, mild cough, and low grade fever before signs and symptoms of upper airway obstruction become apparent. The child then develops the characteristic barking cough, hoarseness and inspiratory stridor.

Croup is a clinical diagnosis and does not require a radiograph of the neck. Radiographs of the neck can show the typical subglotting narrowing, or steeple sign, of croup on the posteroanterior view. “

How well did you know this?
1
Not at all
2
3
4
5
Perfectly
8
Q
"Case of proven untreated syphillic mother with both VDRL and FTA positive. Newborn is asymptomatic and titers are the same or less than four fold increase from mother's. Management?
A. No treatment
B. Pen G IM single dose
C. Pen G IV for 10 days
D. Pen VK for 14 days"
A

C

“Nelson 21st p1597 Fig. 245.10 Algorithm for evaluation and treatment of infants born to mothers with reactive serologic tests for syphilis

Fetal titers are he same or less than fourfold of the maternal titers + asymptomatic - infant has possible congenital syphilis

Nelson 21st p1599
Congenital syphilis is treated with:
1. Aqueous Penicllin G (100,000 - 150,000 units/kg/day BID IV x 1 week, then q8 thereafter)
2. Procaine penicillin G (50,000 units/kg IM OD x 10 days) “

How well did you know this?
1
Not at all
2
3
4
5
Perfectly
9
Q
"True of congenital syphillis
A. Most symptoms occur at 1 yr old
B. Most are asymptomatic at birth
C. Withholding treatment will result to death at 6 months
D. None of the above are true"
A

B

“Nelson 21st p1593
Untreated syphilis during pregnancy results in a vertical transmission rate approaching 100% with profound effects on pregnancy outcome, reflecting obliterating endarteritis. Fetal or perinatal death occurs in 40% of affected infnats.

Most infected infants are asymptomatic at birth, including up to 40% with CSF seeding, and are identified only by routine prenatal screening.

The early signs appear during the first 2 yr of life, and the late signs appear gradually during the first 2 decades. “

How well did you know this?
1
Not at all
2
3
4
5
Perfectly
10
Q
"Why is aspirin contraindicated in dengue. Most dangerous effect
A. Affects hemostasis
B. Can lead to bloody stools
C. Can lead to anaphylaxis
D. None of the above"
A

A

How well did you know this?
1
Not at all
2
3
4
5
Perfectly
11
Q
"When is antibiotics recommended?
A. Croup
B. Laryngitis
C. Epiglottitis
D. Bronchitis"
A

C

“Nelsons 21st p2202-2203
With the exceptions of diphtheria, bacterial tracheitis, and epiglottitis, most acute infections of the upper airway are caused by viruses.

The parainfluenza viruses account for approximately 75% of cases. Other viruses associated with croup include influenza, adenovirus, RSV, and measles.

In the past, H. influenza type b was the most commonly identified etiology of acute epiglottitis. Other agents such as S. pyogenes, S. pneumoniae, nontypeable H. influenza, and S. aureus represent a larger portion of pediatric cases of epiglottitis in vaccinated children.

Nelsons p2206
Bacterial tracheitis is an acute bacterial infection of the upper airway that is potentially life-threatening. S. aureus is the most commonly associated pathogen.

Nelsons 21st p2912
GAS is the most important bacterial cause of acute pharyngitis, but viruses predominate as acute infectious causes of pharyngitis. “

How well did you know this?
1
Not at all
2
3
4
5
Perfectly
12
Q

“When is mumps infectious?
A. 7 days before to 7 days after appearance of the swelling
B. 2-3 days before prodrome and 3 days after swelling
C. 2 days before and after swelling
D. Whole duration”

A

A

“Nelsons 21st p1680
Mumps is spread from person to person by respiratory droplets. Virus appears in the saliva from up to 7 days before to as long as 7 days after onset of parotid swelling. The period of manixmum infectiousness is 1-2 days before to 5 days after onset of parotid swelling. “

How well did you know this?
1
Not at all
2
3
4
5
Perfectly
13
Q
"Varicella infectious in mother to child
A. 5 days before to 2 days after appearance of rash
B. 2 days to 3 days
C. At birth
D. Anytime after"
A

A

“Nelsons 21st p1709-1710
Persons with varicella may be contagious 24-48hr before the rash is evident and until vesicles are crusted, usually 3-7 days after the onset of rash, consistent with evidence that VZV is spread by aerosolization of virus in cutaneous lesions; spread from oropharyngeal secretions may occur but to a much lesser extent

Infants whose mothers demonstrate varicella in the period from 5 days prior to delivery to 2 days afterward are at high risk for severe varicella. “

How well did you know this?
1
Not at all
2
3
4
5
Perfectly
14
Q

“When is Hepa A most infectious
A. 2 weeks before to 7 days after jaundice appears
B. 1 week to 5 days after jaundice appears
C. 1 month before and 1 month after jaundice appears
D. None of the above”

A

A

“Nelsons 21st p2108
HAV is highly contagious. Transmission is almost always by person-to-person through the fecal-oral route.

Patients infected with HAV are contagious for 2wk before and approximately 7 days after the onset of symptoms. “

How well did you know this?
1
Not at all
2
3
4
5
Perfectly
15
Q
"Rabies prophylaxis is in someone with dog abrasions that did not bleed?
A. Rabies vaccine only
B. Rabies vaccine and lg
C. Rabies lg only
D. Coamoxiclav"
A

A

“Fundamentals of Pediatrics vol 1 p638 Table 26-4 Categories of rabies exposure and corresponsing management

Category I

  • Feeding/touching an animal
  • Licking of intact skin
  • Casual contact and routine delivery of health care to patient with signs and symptoms of rabies

Category II

  • Nibbling of uncovered skin with or without bruising/hematoma
  • Minor scratches/abrasions without bleeding
  • Minor scratches/abrasions induced to bleed

Category III

  • Transdermal bites (puncture wounds, lacerations, avulsions) or scratches/abrasions with spontaneous bleeding
  • Licks on broken skin
  • Exposure to a rabies patient through bites, contamination of mucous membranes or open skin liesions with body fluids through splattering and mouth-to-mouth resuscitation
  • Handling of infected caracass or ingestion of raw infected meat
  • All category II exposures on head and neck

Management

  1. Cat I - wash with soap and water, no vaccine or RIG needed; pre-exposure prophylaxis in high risk persons
  2. Cat II - wash with soap and water, start rabies vaccine immediately, RIG not indicated
  3. Cat III - wash with soap and water, start rabies vaccine and RIG immediately “
How well did you know this?
1
Not at all
2
3
4
5
Perfectly
16
Q

“2 yo with cheek lesions that wrinkles and peels when touched

a. SJS
b. TEN
c. SSSS
d. Kawasaki”

A

C

“Nelson 21st p3483 SJS
Cutaneous lesions in SJS generally consist initially of erythematous macules that rapidly and variably develop central necrosis to form vesicles, bullae, and areas of denudation on the face, trunk, and extremities. The skin lesions are acommpanied by involvement of 2 or more mucosal surfaces, namely the eyes, oral cavity, upper airway or esophagus, gastrointestinal tract, or anogenital mucosa

Nelson 21st p3484 TEN
TEN is defined by:
1. Widespread blister formation and morbilliform or confluent erythema, associated with skin tenderness
2. Absence of target lesions
3. Sudden onset and generalization within 24-48hr
4. Histologic findings of full-thickness epidermal necrolysis and a minimal-to-absent dermal infiltrate.

Nelson 21st p3553 SSSS
SSSS, which occurs predominantly in infants and children younger than 5 yr of age, includes a range of disease from localized bullous impetigo to generalized cutaneous involvement with systemic illness.

Scarlatiniform erythema develops profusely and is accentuated in flexural and perorificial areas. The brightly erythematous skin may rapidly acquire a wrinkled appearance, and in severe cases, flaccid blisters and erosions develop profusely. At this stage, areas of the epidermis may separate in response to gentle shear force (Nikolsky sign). As large sheets of epidermis peel away, moist, glistening areas become apparent, initially in the flexures and subsequently over much of the body surface.

Nelson 21st p1310 Kawasaki
In addition to fever, the 5 principal criteria of KD are:
1. Bilateral nonexudative conjunctival injection with limbal sparing
2. Erythema of the oral and pharyngeal mucosa with strawberry tongue and red, cracked lips
3. Edema and erythema of the hands and feet
4. Rash of various forms (maculopapular, erythema multiforme, scarletiniform or less often psoriatic-like, urticarial, or micropustular)
5. Nonsuppurative unilateral cervical lymphadenopathy (>1.5cm) “

How well did you know this?
1
Not at all
2
3
4
5
Perfectly
17
Q
"Mother with this hepatitis has increaed risk for HCC on the newborn
A. Hepa A
B. Hepa B
C. Hepa C
D. Hepa D"
A

B

“Nelson 21st p2114-2115
In general, the outcome after acute HBV infection is favorable, despite the risk of ALF. The risk of developing chronic infection brings the risks of liver cirrhosis and HCC to the forefront. Perinatal transmission leading to chronicity is responsible for the high incidence of HCC in young adults in edemic areas. “

How well did you know this?
1
Not at all
2
3
4
5
Perfectly
18
Q
"Case of stridor, hoarseness and barking paroxysm. Most common cause
A. Parainflunza
B. Hib
C. Strep
D. RSV"
A

A

“Nelson 21st p2203
Most patients have an URTI with some combination of rhinorrhea, pharyngitis, mild cough, and low grade fever before signs and symptoms of upper airway obstruction become apparent. The child then develops the characteristic barking cough, hoarseness and inspiratory stridor.

The parainfluenza viruses account for approximately 75% of cases. Other viruses associated with croup include influenza, adenovirus, RSV, and measles.

Croup is a clinical diagnosis and does not require a radiograph of the neck. Radiographs of the neck can show the typical subglotting narrowing, or steeple sign, of croup on the posteroanterior view. “

How well did you know this?
1
Not at all
2
3
4
5
Perfectly
19
Q
"What vaccine needs a booster at during convalescence since disease does not confer lifelong immunity
A. Diphtheria
B. Pertussis
C. Mumps
D. Varicella"
A

B

“Nelson 21st p1492
Neither natural disease nor vaccination provides complete or lifelong immunity against pertussis reinfection or disease.

Although the DTaP series is protective short-term, vaccine effectiveness wanes rapidly, with estimates of only 10% protection 8.5yr after the 5th dose. Tdap protection is also short-lived, with efficacy falling from >70% initially to 34% within 2-4yr. “

How well did you know this?
1
Not at all
2
3
4
5
Perfectly
20
Q

“A girl from Samar came in for 3-day history of watery diarrhea, crampy abdominal pain. Sister has the same problem. What is the diagnosis?

a) Giardiasis
b) Amoebiasis
c) Cryptosporidiasis
d) Schistosomiasis”

A

C

“Nelson 21st p1836 Cryptosporidium
Cryptosporidium is recognized as a leading protozoal cause of diarrhea in children worldwide and is a common cause of outbreaks in childcare centers.

Diarrhea is initiated by ingestion of infectious oocyts that were ingested in the feces of infected humans and animals.

Cryptosporidium infection is associated with profuse, watery, nonbloody diarrhea that can be accompanied by diffuse crampy abdominal pain, nausea, vomiting, and anorexia

Nelson 21st p1834 Giardia
Most symptomatic patients usually have a limited period of acute diarrheal disease with or without low grade fever, nausea, and anorexia. In an small proportion of patients, an intermittent or more protracted course characterized by diarrhea, abdominal distension and cramps, bloating, malaise, flatulence, nausea, anorexia, and weight loss occurs.

Stools may initially be profuse and watery and later become foul smelling and may flloat. Stools do not contain blood, mucus, or fecal leukocytes. Varying degrees of malabsorption may occur.

Nelson 21st p1832 Amebiasis
The onset of amebic colitis is usually gradual, with colicky abdominal pains and frequent bowel movements (6-8x/day). Diarrhea is frequently associated with tenesmus. Almost all stool is heme-positive, but most patients do not present with greasy bloody stools. Generalized constitutional symptoms and signs are characteristically absent, with fever documented in only 1/3 of patients.

Nelson 21st p1891 Schistosoma
Two main clincal syndromes arise from Schistosoma infection: urogenital schistosomiasis caused by S. hematobium and intestinal schistosomaisis caused by S. mansoni or S. japonicum.

Children with chronic schistosomiasis may have intestinal symptoms; colicky abdominal pain and bloody diarrhea are the most common. However, the intestinal phase may remain subclinical, and the late syndrome of hepatosplenomegaly, portal hypertension, ascites, and hematemesis may be the first clinical presentation. “

How well did you know this?
1
Not at all
2
3
4
5
Perfectly
21
Q

“Another girl from Samar came in for 2-week history of diarrhea, greasy stools, and tenesmus. Stool exam showed no fecal blood, mucus, leukocytes. What is the pathologic agent?

a) Giardia lamblia
b) Entamoeba histolytica
c) Shigella dysenteriae
d) Cryptosporidium”

A

A


Nelson 21st p1836 Cryptosporidium
Cryptosporidium is recognized as a leading protozoal cause of diarrhea in children worldwide and is a common cause of outbreaks in childcare centers.

Diarrhea is initiated by ingestion of infectious oocyts that were ingested in the feces of infected humans and animals.

Cryptosporidium infection is associated with profuse, watery, nonbloody diarrhea that can be accompanied by diffuse crampy abdominal pain, nausea, vomiting, and anorexia

Nelson 21st p1834 Giardia
Most symptomatic patients usually have a limited period of acute diarrheal disease with or without low grade fever, nausea, and anorexia. In an small proportion of patients, an intermittent or more protracted course characterized by diarrhea, abdominal distension and cramps, bloating, malaise, flatulence, nausea, anorexia, and weight loss occurs.

Stools may initially be profuse and watery and later become foul smelling and may flloat. Stools do not contain blood, mucus, or fecal leukocytes. Varying degrees of malabsorption may occur.

Nelson 21st p1832 Amebiasis
The onset of amebic colitis is usually gradual, with colicky abdominal pains and frequent bowel movements (6-8x/day). Diarrhea is frequently associated with tenesmus. Almost all stool is heme-positive, but most patients do not present with greasy bloody stools. Generalized constitutional symptoms and signs are characteristically absent, with fever documented in only 1/3 of patients.

Nelson 21st p1509 Shigella
Bacillary dysentery is clinically similar regardless of infecting serotype. The diarrhea may be watery and of large volume initially, evolving into frequent, small-volume, bloody mucoid stools. “

How well did you know this?
1
Not at all
2
3
4
5
Perfectly
22
Q

A chronic complication of measles

A

SSPE

“Nelson 21st p1673-1674
Subacute sclerosing panencephalitis (SSPE) is a chronic complication of measles with a delayed onset and an outcome that is nearly always fatal.

The diagnosis of SSPE can be eastablished through documentation of a complatible clinical course and at least one of the following supporting findings:
1. Measles antibody detected in CSF
2. Characteristic EEG findings
3. Typical histologic findings in and/or isolation of virus or viral antigen from brain tissue obtained by biopsy or postmortem examination

How well did you know this?
1
Not at all
2
3
4
5
Perfectly
23
Q

“Most devastating copious watery diarrhea.

a) Cholera
b) ETEC
c) EHEC
d) Giardia”

A

A

“Nelson 21st p1516
Following an incubation period of 1-3 days, acute watery diarrhea and vomiting ensue. Diarrhea can progress to painless puring of profuse rice-water stools (suspended flecks of mucus) with a fishy smell, which is the hallmark of the disease.

Cholera gravis, the most severe form of the disease, results when puring rates of 500-1000 ml/hr occur. The purging leads to dehydration manifested by decreased urine output, sunken fontanel, sunken eyes, absence of tears, dry oral mucosa, shriveled hands and feet, poor skin turgor, thready pulse, tachycardia, hypotension, and vascular collapse.
Although patients may initially be thirsty and awake, they rapidly progress to obtundation and coma. If fluid losses are not rapidly corrected, death can occur within hours. “

How well did you know this?
1
Not at all
2
3
4
5
Perfectly
24
Q

Infection that initially presents with acute fever, then rapidly progresses to shock and purpura.

A

acute meningococcemia

“Nelson 21st p1471-1472
THe most common form of meningococcal infection is asymptomatic carriage of the organism in the nasopharynx. In the rare cases where invasive disease occurs, the clinical spectrum of meningococcal disease varies widely, but the highest proportion of cases present with meningococcal meningitis (30-50%)

Acute meningococcal septicemia cannot be distinguished from other viral or bacterial infections early after onset of symptoms. Typical nonspecific early symptoms include fever, irritability, lethargy, respiratory symptoms, refusal to drink, and vomiting.

As disease progresses, cold hands or feet and abnormal skin color may be important signs, capillary refill time becomes prolonged, and a nonblanching or petechial rash will develop in >80% of cases. In fulminant meningococcal septicemia, the disease progresses rapidly over several hours from fever with nonspecific signs to septic shock characterized by prominent petechiae and purpura (purpura fulminans) with poor peripheral perfusion, tachycardia (to compensate for reduced blood volume resulting from capillary leak), increased respiratory rate (to compensate for pulmonary edema), hypotension (a late sign of shock in young children), confusion, and coma (resulting from decreased cerebral perfusion). Coagulopathy, electrolyte disturbance (esp. hypokalemia), acidosis, adrenal hemorrhage, renal failure, and myocardial failure may develop. Meningitis may be present. “

How well did you know this?
1
Not at all
2
3
4
5
Perfectly
25
Q

“Patient came in with nodules on the shin. These lesions are also found in patients infected with Group A strep and Inflammatory Bowel Disease?

a) Erythema multiforme
b) Erythema nodosum
c) Erythema marginatum
d) Erythema toxicum”

A

B

How well did you know this?
1
Not at all
2
3
4
5
Perfectly
26
Q

“Proven causal relationship in vaccination

a) MMR and autism
b) injection-related events and syncope
c) BCG and pneumonia
d) None of the above”

A

B

How well did you know this?
1
Not at all
2
3
4
5
Perfectly
27
Q

“3 yr old male was seen at the ER for high fever and difficulty in breathing. He was noted to have drooling, and was sitting upright with neck hyperextended.

a) epiglottitis
b) acute pharyngitis
c) diphtheric pharyngit is
d) laryngotracheobronchitis”

A

A

“Nelson 21st p2203
Epiglottitis is characterized by an acute rapidly progressive and potentially fulminating course of high fever, sore throat, dyspnea, and rapidly progressing respiratory obstruction…Within a matter of hours, the patient appears toxic, swallowing is difficult, and breathing is labored. Drooling is usually present, and the neck is hyperextended in an attempt to maintain the airway. The child may assume the tripod position, sitting upright and leaning forward with the chin up and mouth open while bracing the arms. The diagnosis requires visualization under controlled circumstances of a large, cherry red, swollen epiglottis by laryngoscopy. “

How well did you know this?
1
Not at all
2
3
4
5
Perfectly
28
Q

“Most common cause of watery diarrhea worldwide.

a) Rotavirus
b) ETEC
c) Cholera
d) Giardia”

A

A

“Nelson 21st p2012
Rotavirus is the most common cause of AGE among children throughout the world. Several other viruses occur less frequently: Norovirus, sapovirus, adenovirus. “

How well did you know this?
1
Not at all
2
3
4
5
Perfectly
29
Q

“Patient presented with pink macules that progress to hemorrhagic nodules and eventually to ulcers with ecchymotic and gangrenous centers with eschar formation.

a) Pseudomonas aeruginosa
b) Staphylococcus aureus
c) Streptococcus pyogenes
d) Fusobacterium necrophorum”

A

A

“Nelson 21st p1529
The characteristic skin lesions of P. aeruginosa, ecthyma gangrenosum, whether caused by direct inoculation or a metastatic focus secondary to septicemia, begin as pink macules and progress to hemorrhagic nodules nd eventually to ulcers with ecchymotic and gangrenous centers with eschar formation, surrounded by an intendse red areola. “

How well did you know this?
1
Not at all
2
3
4
5
Perfectly
30
Q

“A 2-year old boy was seen at the ER with 3-dayhistory of cough and fever. He is noted to have inspiratory wheezing and fine crackles.

a) pneumonia
b) bronchiolitis
c) bronchitis
d) tracheitis”

A

C

“Nelson 21st p2220 Acute bronchitis
The child first presents with nonspecific upper respiratory infectious symptoms such as rhinitis. 3 to 4 days later, a frequent dry hacking cough develops, which may or may not be productive. After several days the sputum can become purulent. Chest pain may be a prominent complaint in older children and is exacerbated by coughing.

Early findings include no or low grade fever and upper respiratory signs such as nasopharyngitis, conjunctivitis, and rhinitis. Auscultation of the chest may be unremarkable at this early phase. As the syndrome progresses and cough worsens, breath sounds become coarse, with coarse and fine crackles and scattered high pitched wheezing. CXR is normal or can have increased bronchial markings

Nelson 21st p2219 Acute bronchiolitis
Acute bronchiolitis is usually preceded by exposure to contacts with a minor respiratory illness within the prevous week. The infant first develops signs of upper respiratory tract infection with sneezing and clear rhinorrhea. This may be accompanied by diminished appetite and fever. Gradually, respiratory distress ensues, with paroxymal cough, dypnea, and irritability.

The physical exam is often dominated by wheezing and crackles. Expiratory time may be prolonged. Work of breathing may be markedly increased, with nasal flaring and retractions.

Nelson 21st p2206 Tracheitis
Typically the child has a brassy cough, apparently as part of a viral laryngotracheobronchitis. High fever and toxicity with respiratory distress can occur immediately or after a few days of apparent improvement. The patient can lie flat, does not drool, and does not hav the dysphagia assocaited with epiglottitis.

The major pathologic feature appears to be mucosal swelling at the level of the cricoid cartilage, complicated by copious thick purulent secretions, sometimes causing pseudomembranes.

Nelson 21st p2269
Pneumonia is frequently preceded by several days of symptoms of an upper respiratory tract infection, typically rhinitis and cough. Tachypnea is the most consistent clinical manifestation of pneumonia.

Increased work of breathing accompanied by intercostal, subcostal, and suprasternal retractions, nasal flariing, and use of accessory muscles is common. Auscultation of the chest may reveal crackles and wheezing. “

How well did you know this?
1
Not at all
2
3
4
5
Perfectly
31
Q

“One of the causes of moderate to severe diarrhea, where WHO patterned management for diarrhea.

a) Vibrio cholera
b) Rotavirus
c) ETEC
d) Giardia”

A

A

“Nelson 21st p1516
Following an incubation period of 1-3 days, acute watery diarrhea and vomiting ensue. Diarrhea can progress to painless puring of profuse rice-water stools (suspended flecks of mucus) with a fishy smell, which is the hallmark of the disease.

Cholera gravis, the most severe form of the disease, results when puring rates of 500-1000 ml/hr occur. The purging leads to dehydration manifested by decreased urine output, sunken fontanel, sunken eyes, absence of tears, dry oral mucosa, shriveled hands and feet, poor skin turgor, thready pulse, tachycardia, hypotension, and vascular collapse.
Although patients may initially be thirsty and awake, they rapidly progress to obtundation and coma. If fluid losses are not rapidly corrected, death can occur within hours. “

How well did you know this?
1
Not at all
2
3
4
5
Perfectly
32
Q

“Most common bacterial isolate in an infected dog-bitten wound.

a) Gram negative
b) Tetanus
c) S. pneumoniae
d) S. aureus”

A

D

“Nelson 21st p3819
The predominant bacterial species isolated from infected dog bite wounds are S. aureus (20-30%), Pasteurella multocida (20-30%), Staphylococcus intermedius (25%) and Capnocytophaga canimorsus; approximately 50% of dog bite wound infections also contain mixed anaerobes

Similar species are isolated from infected cat bite wounds, however Pasteurella multocida is the predominant species in at least 50% of cat bite wound infections.

At least 50% of rats harbor strains of Streptobacillius miniliformis in the oropharynx, and approximately 25% harbor Spirillum minor, an anareobic gram negative organism.

In human bite wounds, nontypeable strains of H. influenzae, Eikenella corrodens, S. aureuus, alpha hemolytic streptococci, and B-lactamase producing aerobes (~50%) are the predominant species. Clenched fist injuries are particularly prone to infection by Eikenella (25%) and anaerobic bacteria (50%)”

How well did you know this?
1
Not at all
2
3
4
5
Perfectly
33
Q

“Which vaccine would you NOT give in a patient undergoing chemotherapy?

a. IPV
b. Varicella
c. Influenza
d. Tdap”

A

B

Live attenuated vaccines are contraindicated in immunocompromised patients and patients undergoing chemotherapy

How well did you know this?
1
Not at all
2
3
4
5
Perfectly
34
Q

“A child has recurrent pneumonia treated with amoxicillin, cephalosporin etc. On history the patient was noted not to be
compliant with medications. Now admitted, Chest x- ray showed lobar consolidation of the right lung. What is the antibiotic of choice
a. Ceftriaxone
b. Linezolid
c. Vancomycin
d. Meropenem”

A

A

“PAPP 2021 PCAP CPG p32
Starting with broad spectrum antibiotics to treat uncomplicated PCAP is highly discouraged and such antibiotics should be reserved for more complicated forms of the disease and for drug-resistant pathogents.

Amoxicillin is still the treatment of choice because it is effective against the majority of pathogens causing PCAP in this age group. High dose amoxicillin is recommended for treatment of suspected or confirmed penicillin-resistant S. pneumonia; the resistance of which can be overcome at higher drug concentrations.

Because the pharmacokinetics of oral cephalosporins are far inferior to amoxicillin, their use in PCAP should be reserved for patients who are allergic to penicillin or patients with isolates known to be resistant to amoxicillin but sensitive to cephalosporins. “

How well did you know this?
1
Not at all
2
3
4
5
Perfectly
35
Q

“In a patient with pneumonia, What is the drug of choice in a child who is not allergic to penicillin?

a. Amoxicillin
b. Azithromycin
c. Cefuroxime
d. Ceftriaxone”

A

A

”"”PAPP 2021 PCAP CPG p32
Starting with broad spectrum antibiotics to treat uncomplicated PCAP is highly discouraged and such antibiotics should be reserved for more complicated forms of the disease and for drug-resistant pathogents.

Amoxicillin is still the treatment of choice because it is effective against the majority of pathogens causing PCAP in this age group. High dose amoxicillin is recommended for treatment of suspected or confirmed penicillin-resistant S. pneumonia; the resistance of which can be overcome at higher drug concentrations.

Because the pharmacokinetics of oral cephalosporins are far inferior to amoxicillin, their use in PCAP should be reserved for patients who are allergic to penicillin or patients with isolates known to be resistant to amoxicillin but sensitive to cephalosporins. “””

How well did you know this?
1
Not at all
2
3
4
5
Perfectly
36
Q

“2nd to rotavirus, which is the next most prevalent cause of diarrhea in the Philippines?

a. ETEC
b. EHEC
c. Shigella
d. E. histolytica”

A

A

“Fundamentals of Pediatrics vol 2 p1296
Rotavirus is the leading cause of severe watery diarrhea globally and is responsible for 527,000 deaths annually. It counts for 29% of diarrheal diseases among children under 5 years old.

E. coli is the cause of 25% of diarrheas in the developing countries. Of the 5 groups of E. coli, the most common agent is ETEC , which causes acute water diarrhea in developing countries.

Shigella is the cause of 10-15% of acute diarrheas in children under 5 years. Known as the prototype organism causing bloody diarrhea, it is the most common cause of dysentery in children.”

How well did you know this?
1
Not at all
2
3
4
5
Perfectly
37
Q

“Child with peri-anal pruritus at night?

a. E. vermicularis
b. Ascaris lumbricoides
c. Trichuris trichuira
d. Hookworm”

A

A


Nelson 21st p1882
Pinworm infection is innocuous and rarely causes serious medical problems. The most common complaints include itching and restless sleep secondary to nocturnal perianal or perineal pruritis. “

How well did you know this?
1
Not at all
2
3
4
5
Perfectly
38
Q

“A teenager went camping with friends and went into caves. Came back with pulmonary symptoms (cough), hepatomegaly etc. Diagnosis?

a. Histoplasmasmosis
b. HIV
c. Tuberculosis
d. Coccidiomycosis”

A

A

“Nelson 21st p1650 Histoplasmosis
H. capsulatum thrives in soil rich in nitrates such as areas that are heavily contaminated with bird or bat droppings or decayed wood. Focal outbreaks of histoplasmosis have been reported after intense exposure to bat guano in caves and along bridges frequented by bats.

Acute pulmonary histoplasmosis follows initial or recurrent respiratory exposure to microconidia. The prodrome is not specific and usually consists of flu-like symptoms, including headache, fever, chest pain, cough, and myalgias. Hepatosplenomegaly occurs more often in infants and young children. Symptomatic infections may be associated with significant respiratory distress and hypoxia and may require intubation, mechanical ventilation, and steroid therapy.

Nelson 21st p1654-1655 Coccidiomycosis
Coccidioides spp. inhabit soil in arid regions in the USA, Mexico, and South America. Incidence increases during windy, dry periods that follow rainy seasons. Seismic events, archeologic excavations, and other activities that disturb contaminated sites have caused outbreaks.

The clinical constellation of erythema nodosum, fever, chest pain, and arthralgias (esp. knees and ankles) has been termed desert rheumatism and valley fever. “

How well did you know this?
1
Not at all
2
3
4
5
Perfectly
39
Q

“A child with microcytic hypochromic anemia was on iron for 3 months. Repeat CBC still showed anemia. Stool exam was done and showed Necator americanus. What is the pathophysiology of anemia?

a. Rupture of the capillaries
b. Ingestion of blood by the parasite
c. Blood loss in stool
d. Attachment of the parasite to the GI tract”

A

A

“Nelson 21st p1880
The major morbidity of human hookworm infection is a direct result of intestinal blood loss. Adult hookworms adhere tenaciously to the mucosa and submucosa of the proximal small intestine by using their cutting plates or teeth and a muscular esophagus that creates negative pressure in their buccal capsules.

At the attachment site, host inflammation is downregulated by the release of anti-inflammatory polypeptides by the hookworm. Rupture of capillaries in the lamina propria is followed by blood extravasation, with some of the blood ingested directly by the hookworm. After ingestion, the blood is anticoagulated, the RBCs are lysed, and the hemoglobin released and digested.

Each adult A. duodenale hookworm causes loss of an estimated 0.2ml of blood per day; blood loss is less for N. americanus. “

How well did you know this?
1
Not at all
2
3
4
5
Perfectly
40
Q

“A history of which condition is a clue to diagnose a child with chronic regional lymphadenopathy?

a. Cat scratch disease
b. EBV
c. TB adenitis
d. Leukemia”

A

A

“Nelson 21st p1540-1541
The most common presentation of Bartonella infection is cat-scratch disease, which is a subacute, regional lymphadenitis caused most frequently by B. henselae. It is the most common cause of chronic lymphadenitis that persists for >3wk

After an incubation period of 7-12 days. 1 or more 3-5mm red papules develop at the site of cutanous inoculation, often reflecting a linear cat scratch.

Lymphadenopathy is generally evident within 1-4 wk. Chronic regional lymphadenitis is the hallmark, affecting the 1st or 2nd set of nodes draining the entry site. Affected lymph nodes in order of frequency include the axillary, cervical, submandibular, preauricular, epitrochlear, femoral, and inguinal nodes. “

How well did you know this?
1
Not at all
2
3
4
5
Perfectly
41
Q

“Case about sinusitis. What is the 1st line drug of choice?

a. Amoxicillin
b. Co- amoxiclav
c. Clindamycin
d. Cefuroxime”

A

A

“Nelson 21st p2190
Initial therapy with amoxicillin (45mkd BID) is adequate for most children with uncomplicated mild to moderate severity acute bacterial sinusitis. Alternative treatments for the penicillin-allergic patients include cefdinir, cefuroxime, cefpodoxime, or cefixime. In older children, levofloxacin is an alternative antibiotic. “

How well did you know this?
1
Not at all
2
3
4
5
Perfectly
42
Q
"Case: epiglotitittis. Agent?
a. Influenza
b. Parainfluenza1,2,3
C. Hib
d. RSV"
A

C

“Nelsons 21st p2202-2203
With the exceptions of diphtheria, bacterial tracheitis, and epiglottitis, most acute infections of the upper airway are caused by viruses.

The parainfluenza viruses account for approximately 75% of cases. Other viruses associated with croup include influenza, adenovirus, RSV, and measles.

In the past, H. influenza type b was the most commonly identified etiology of acute epiglottitis. Other agents such as S. pyogenes, S. pneumoniae, nontypeable H. influenza, and S. aureus represent a larger portion of pediatric cases of epiglottitis in vaccinated children. “

How well did you know this?
1
Not at all
2
3
4
5
Perfectly
43
Q

“Child previously had varicella. After several weeks the patient would fall to one side, difficult walking. Diagnosis?

a. Cerebellar ataxia
b. Encephalitis
c. Friedrichs ataxia
d. Medulloblastoma”

A

A

“Nelsons 21st p1712
Encephalitis and acute cerebellar ataxia are well-described neurologic complications of varicella. Morbidity from CNS complications is highest among patients younger than 5yr and older than 20yr.

Nuchal rigidity, altered consciousness, and seizures characterize meningoencephalitis. Patient with cerebellar ataxia have a gradual onset of gait disturbance, nystagmus, and slurred speech.

Neurologic symptoms usually begin 2-6 days after the onset of the rash but may occur during the incubation period or after resolution of the rash. “

How well did you know this?
1
Not at all
2
3
4
5
Perfectly
44
Q

“Patient traveled to malaria endemic area (Mindoro), came back after 16 days with fever every 48 hours with defervescence and fatigue in between. Diagnosis:

a. Malariae
b. Ovale
c. Vivax
d. Falciparum”

A

C

"Incubation period 
Falciparum 9-14 days
Vivax 12-17 days
Ovale 16-18 days
Malariae 18-40 days 

Fever pattern
Vivax, ovale - every other day
Malariae - every 3rd day
Falciparum - periodicity less apparent

Plasmodium ovale least common, primarily africa
Plasmodium vivax found in SE Asia”

How well did you know this?
1
Not at all
2
3
4
5
Perfectly
45
Q

“Preterm neonate, 1 month at the NICU, noted to have decreased tone and apnea. The patient has a central line with CONS. How to definitively diagnose CONS?

a. Hx of prolonged nicu stay, presence of new signs and symptoms and previous iv antibiot ic tx
b. Culture IV site
c. Blood culture showed S. epidermidis after 48 hours
d. Blood culture central and peripheral line and then start antibiotics”

A

D

“Nelson 21st p1436
True bacteremia should be suspected if blood cultures grow rapidly (within 24hr), >1 blood culture is positive with the same CONS strain, cultures from both line and peripheral sites are positive, and clinical and laboratory signs and symptoms compatible with CONS sepsis are present and subsequently resolve with appropriate therapy.

Before initiating presumptive antibiotic therapy in such patients, it is always prudent to draw 2 separate blood cultures to facilitate subsequent interpretation if CONS is grown.

Nelson 21st p1411
Blood cultures collected before beginning antibiotic therapy are gneerally positive from both the CVC and the peripheral blood. It is important not to collect cultures unless infection is suspected, as blood culture contamination may occur and lead to inappropriate therapy. To help interpret positive cultures with common skin contaminants, blood cultures should be collected from at least 2 sites, preferably including all lumens of CVC and the peripheral blood, before intiation of antibiotic therapy. “

How well did you know this?
1
Not at all
2
3
4
5
Perfectly
46
Q

“A 5 year old child, previously well suddenly had changes in behavior, irritability plus choreoathetosis. What infection is important to know in the history?

a. Rubeola
b. Enterovirus
c. Herpes
d. Varicella”

A

C

“Nelson 21st p1705
HSV encephalitis is the leading cause of sporadic, nonepidemic encephalitis in children and adults in the US. It is an acute necrotizing infection generally involving the frontal and/or temporal cortex and limbic system and, beyond the neonatal period, is almost always caused by HSV-1.

The infection may manifest as nonspecific findings, including fever, headache, nuchal rigidity, nausea, vomiting, generalized seizures, and alteration of consciousness. Injury to the frontal or temporal cortex or limbic system may produce findings more indicative of HSV encephalitis, including anosmia, memory loss, peculiar behavior, expressive aphasia and other changes in speech, hallucinations, and focal seizures.

Nelson 21st p3182 Anti-N-methyl-D-aspartate receptor encephalitis

In a small number of patients, anti-NMDAR encephalitis occurs simultaneously with or after infection with a variety of pathogens, including Mycoplasma pneumoniae, HSV1, HHV6, enterovirus, and influenza virus.

There is evidence that some patients with HSV encephalitis develop antibodies aginst the GluN1 subunit of the NMDAR and other neuronal cell surface proteins and receptors, which leads to the presentation of new or relapsing neurologic symptosm 2-12 wk after completing treatment for HSV encephalitis.

In children younger than 4yr, this type of autoimmune encephalitis usually manifests with choreoathetosis and dyskinesias (known as choreoathetosis post-HSV encephalitis). In contrast, older children and adults more often develop predominantly behavioral symptoms. “

How well did you know this?
1
Not at all
2
3
4
5
Perfectly
47
Q

“True regarding gonococcal infection in children?

a. Gram stain of the urethral discharge shows intracellular gram negative organisms seen equally in males and females
b. Males with the disease will not resolve if not treated
c. 1 dose of ceftriaxone is needed to cure neonatal disease
d. None of the above”

A

C

“Nelson 21st p1481
In males with symptomatic urethritis, a presumptive diagnosis of gonorrhea can be made by identification of gram negative intracellular diplococci (within leukocytes) in the urethral discharge. A similar finding in females is not sufficient because Mima polymorpha and Moraxella, which are normal vaginal flora, have a simlar appearance. The sensitivity of the Gram stain for diagnosing gonococcal cervicitis and asymptomatic infections is also low.

Nelson 21st p1480
Urethritis is usually characerized by a purulent discharge and by dysuria without urgency or frequency. Untreated urethritis in males resolves spontaneously in several weeks or may be complicated by epididymitis, penile edema, lymphangitis, prostatitis, or seminal vesiculitis.

Nelson 21st p1482 Table 219.1 Recommended treatment of gonococcal infections

Neonates

  • Ophthalmia neonatorum: Ceftriaxone IM SD
  • Disseminated infection, scalp abscess, septic arthritis: Ceftriaxone OR cefotaxime IV x 7 days
  • Meningitis: Ceftriaxone or cefotaxime IV x 10-14 days
  • Endocarditis: Ceftriaxone or cefotaxime IV x 28 days minimum”
How well did you know this?
1
Not at all
2
3
4
5
Perfectly
48
Q

“Up until when is Hib vaccine given?

a. 12 months
b. 24 months
c. 48 months
d. 60 months”

A

D

“Prev Ped 2018 p72
Hemophilus influenza type B conjugate vaccine (Hib)
- Given IM
- Indicated for children aged 12-59mos
a. unimmunized or with one Hib vaccine dose vaccine dose recieved before age 12 months, given 2 additional doses 8 weeeks apart
b. With >=2 Hib vaccine doses recieved before age 12 months, give 1 additional dose “

How well did you know this?
1
Not at all
2
3
4
5
Perfectly
49
Q

“Baby with previous antibiotic use. Now with satellite lesions in the diaper area. Treatment?

a. Topical clotrimazole
b. 1% hydrocortisone ointment
c. Mupirocin ointment
d. Oral azole”

A

A

“Nelson 21st p3564
Candidal diaper dermatitis is an ubiquitous problem in infants and, although relatively benign, is often fustrating because of its tendency to recur. Predisposed infants usually carry C. albicans in their intestinal tracts, and the warm, moist, occluded skin of the diaper airea provides an optimal environment for its growth. A seborrheic, atopic, or irritant contact dermatitis usually provides a portal of entry for the yeast.

The primary clinical manifestation consists of an intensely erythematous confluent plaque with a scalloped border and a sharply demarcated edge. It is formed byt he confluence of numerous plaques and vesicular pustules. Satellite pustules, those that stud the contiguous skin, are a hallmark of localized candidal infection. The perianal skin, inguinal folds, perineum, and lower abdomen are usually involved.

Treatment consists of an imidazole cream 2 times daily. The combination of a corticosteroid and an antifungal agent may be justified if inflammation is severe but may confuse the situation if diagnosis is not firmly established. Corticosteroid should not be continued for more than a few days. Protection of the diaper area by an application of a thick zinc oxide paste overlying the candidal preparation may be helpful.

How well did you know this?
1
Not at all
2
3
4
5
Perfectly
50
Q

“Case of adolescent with peritonsillar cellulitis. Which is true?

a. Abscess is prone to rupture with risk of aspiration
b. Surgically drain and give antibiotics
c. Draining does not help in the disease, do tonsillectomy instead
d. Do incisional drainage to isolate GABHS”

A

B

“Nelson 21st p2198
Peritonsillar cellulitis and/or abscess is caused by bacterial invasion through the capsule of the tonsil, leading to cellulitis and/or abscess formation in the surrounding tissues.

The typical patient is an adolescent with a recent history of acute pharygotonsillitis. Clinical manifestations include sore throat, fever, trismus, muffled or garbled voice, and dysphagia. Physical examination reveals an asymmetric tonsillar bulge with displacement of the uvula (diagnostic).

Group A streptococci and mixed oropharyngeal anaerobes are the most common pathogens, with more than four bacterial isolates per abscess typically recovered by needle aspiration.

Treatment includes surgical drainage and antibiotic therapy effective against group A streptococci and anaerobes. Surgical drainage may be accomplished through needle aspiration, incision and drainage, or tonsillectomy.

Tonsillectomy should be considered if there is failure to improve within 24hr of antibiotic therapy and needle aspiration, history of recurrent peritonsillar abscess or recurrent tonsillitis, or complications from peritonsillar abscess

The feared, albeit rare, complication is rupture of the abscess with resultant aspiration pneumonitis. “

How well did you know this?
1
Not at all
2
3
4
5
Perfectly
51
Q

“Mother is 37 weeks pregnant. 3-year-old daughter contracted hand foot mouth disease. What will you advise?

a. Isolate the mother from the family
b. Give mother methisiprinol
c. Let the mother reach term so she can pass the antibodies to the newborn
d. None of the above”

A

A

“Nelson 21st p1692
Hand-foot-and-mouth disease is most frequently caused by coxsackievirus A16, sometimes in large outbreaks, and can also be caused by coxsackievirus A71.

It is usually a mild illness, with or without low grade fever. When the mouth is involved, the oropharynx is inflamed and often contains scattered, painful vesicles on the tongue, buccal mucosa, posterior pharynx, palate, gingiva, and/or lips. These may ulcerate, leaving 4-8mm shallow lesions with surrounding erythema.

Maculopapular, vesicular and/or pustular lesions may occur on the hands and fingers, feet, and buttocks and groin. Skin lesions occur mmore commonly on the hands than feet and are more common on dorsal surfaces, but frequently also affect palms and soles. Hand and feet lesions are usually tender 3-7mm vesicles that resolve in about 1 wk.

Nelson 21st p1697
Pregnant women near term should avoid contact with individuals ill with possible enterovirus infections. If a pregnant woman experiences a suggestive illness, it is advisable not to proceed with emergency delivery unless there is concern for fetal compromise or obstetric emergencies cannot be excluded. Rather, it may be advantageous to extend pregnancy, allowing the fetus to passively acquire protective antibodies. “

How well did you know this?
1
Not at all
2
3
4
5
Perfectly
52
Q

“Patient with ascariasis, suddenly had difficulty in breathing. What happened?

a. Ingestion of worm
b. Aspiration of worm
c. Worm migrated to lungs
d. None of the above”

A

C

“Nelson 21st p1877 Ascariasis
The most common clinical problems are from pulmonary disease and obstruction of the intestinal or biliary tract.

The pulmonary manifestation resemble Loeffler syndrome and include transient respiratory symptoms such as cough and dyspnea, pulmonary infiltrates, and blood eosinophilia.

A more serious complication occurs when a large mass of worms leads to acute bowel obstruction. Ascaris worms also occasionally migrate into the biliary and pancreatic ducts, where they cause cholecystitis or pancreatitis. “

How well did you know this?
1
Not at all
2
3
4
5
Perfectly
53
Q

“16 year old who ate a hamburger, green stool, mucoid, blood tinged, abdominal pain.

a. E. histolytica
b. ETEC
c. EPEC
d. Shigella”

A

D

“Nelson 21st p1509 Shigella
Bacillary dysentery is clinically similar regardless of infecting serotype. Ingestion of shigellae is followed by an incubation period of 12hr to several days before symptoms ensue. Severe abdominal pain, emesis, anorexia, generalized toxicity, urgency, and painful defecation characteristically occur. The typically high fever with shigellosis distinguishes it from EHEC.

The diarrhea may be watery and of large volume initially, evolving into frequent, small volume, bloody mucoid stools. Significant dehydration is related to the fluid losses in feces and emesis.

Nelson 21st p1512 ETEC
The typical signs and symptoms include explosive watery, nonmucoid, nonbloody diarrhea, abdominal pain, nausea, vomiting, and little or now fever. The illness is usually self-limited and resolves in 3-5 days but can occasionally last >1wk

Nelson 21st p1513 EPEC
EPEC causes acute, prolonged, and persistent diarrhea, primarily in children <2 yr old in developing countries. Profuse watery, nonbloody diarrhea with mucus, vomiting, and low-grade fever are common symptoms. Prolonged diarrhea (>7d) and persistent diarrhea (>14d) can lead to malnutrition.

Nelson 21st p1832 Amebiasis
The onset of amebic colitis is usually gradual, with colicky abdominal pains and frequent bowel movements (6-8x/day). Diarrhea is frequently associated with tenesmus. Almost all stool is heme-positive, but most patients do not present with greasy bloody stools. Generalized constitutional symptoms and signs are characteristically absent, with fever documented in only 1/3 of patients.

How well did you know this?
1
Not at all
2
3
4
5
Perfectly
54
Q

“Why no need to treat salmonella with antibiotics

a. Resolves spontaneously
b. Prolongs salmonella excretion
c. Emergence of resistance
d. None of the above”

A

B

“Why no need to treat salmonella with antibiotics

a. Resolves spontaneously
b. Prolongs salmonella excretion
c. Emergence of resistance
d. None of the above”

How well did you know this?
1
Not at all
2
3
4
5
Perfectly
55
Q

“Phase of Viral shedding in measles

a. Incubation
b. Prodromal
c. Exanthematous
d. Recovery “

A

B

“Nelsons 21st p1670
Measles infection consists of 4 phases: incubation period, prodromal illness, exanthematous phase, and recovery.

During incubation, measles virus migrates to regional lymph nodes. A primary viremia ensues that disseminates the virus to the reticuloendothelial system. A secondary viremia spreads the virus to body surfaces.

The prodromal illness begins after the secondary viremia and is associated with epithelial necrosis and giant cell formation in body tissues. Cells are killed by cell-to-cell plasma membrane fusiion associated with viral replication. Virus shedding begins in the prodromal phase

With onset of the rash, antibody production begins, and viral replication and symptoms begin to subside. “

How well did you know this?
1
Not at all
2
3
4
5
Perfectly
56
Q
"Associated with favorable outcome in tetanus
A. Long incubation period
B. Presence of fever
C. Onset of trismus <7 days
D. Tetanic spasms"
A

A

“Nelson 21st p1552
The most important factor that influences outcome is the quality of supportive care. Mortality is highest in very young and very old patients.

A favorable prognosis is associated with a long incubation period, absence of fever, and localized disease.

An unfavorable prognosis is associated with onset of trismus <7 days after injury and onset of generalized tetanic spasms <3 days after onset of trismus. “

How well did you know this?
1
Not at all
2
3
4
5
Perfectly
57
Q
"Associated with fulminant/ poorer prognosis in meningococcemia
A. Purpura fulminans
B. Leukopenia
C. Low or normal ESR
D. seizure"
A

C

“Nelson 21st
Poor prognostic factors on presentation include hypothermia or extreme hyperpyrexia, hypotension or shock, purpura fulminans, seizures, leukopenia, thrombocytopenia (including DIC), acidosis, and high circulating levels of endotoxin and TNF-α. The presence of petechiae for <12 hr before admission, absence of meningitis, and low or normal ESR indicate rapid, fulminant progression and poorer prognosis”

How well did you know this?
1
Not at all
2
3
4
5
Perfectly
58
Q
"Treatment of a child with cough, in distress, with dehydration, given Hib vaccine. Treatment of choice?
A. Amoxicillin
B. Penicillin G 
C. Ampicillin
D. Co-amoxiclav"
A

B

“Diagnosis: PCAP C

PAPP 2021 PCAP CPG p28

For patients classified as having non-severe PCAP, regardless of
immunization status against S. pneumonia and H. influenza, any of the following is considered:
1. Amoxicillin trihydrate x 7 days
2. Amoxicillin-clavulanate x 5-7 days OR cefuroxime x 7 days in settings with documented high level penicillin resistant pneumococci or B-lactamase producing Hib

For patients classified as having severe PCAP, regardless of immunization status against S. pneumonia, any of the following is considered:

  1. Penicillin G q6 if with complete Hib vaccination OR Ampicillin q6 if none, incomplete or unknown Hib vaccination
  2. Cefuroxime q8 OR ceftriaxone q12 to q24 OR ampicillin sulbactam in settings with documented high level penicillin resistant pneumococci or B-lactamase producing Hib
  3. Add clindamycin q6 to q8 when staphylococcal pnuemonia is highly suspected based on clinical and CXR features
  4. In cases of severe and life threatening conditions such as sepsis or shock, vancomycin q6 to q8 is preferred”
How well did you know this?
1
Not at all
2
3
4
5
Perfectly
59
Q
"A case of an infant with respiratory distress with fever, IC retractions, etc
A. PCAP A
B. PCAP B
C. PCAP C
D. None of the above"
A

C

“PAPP 2021 PCAP CPG p15-16
Criteria for severe pneumonia (PCAP C or D requiring admission)

  1. Respiratory signs - cyanosis/hypoxemia, head bobbing, retractions, apena, grunting
  2. CNS signs - lethargic/stuporous/comatose/GCS <13, seizures
  3. Circulatory signs - capillary refill >3s or shock, pallor
  4. General considerations - moderate to severe malnutrition, refusal or inability to take food/drink per orem, some to severe dehydration, age <6 months
  5. Ancillary parameters - CXR or UTZ findings of consolidation, multifocal disease, moderate to large effusion, abscess, air leak; sustained O2 sat at room air <=93% “
How well did you know this?
1
Not at all
2
3
4
5
Perfectly
60
Q
"Preferred diagnostic test for HIV suspect below 18 months
A. HIV RNA PCR
B. HIV DNA PCR
C. HIV Culture
D. HIV p24"
A

A

“HIV RNA used in the Philippines

Nelson 21st p1787
Viral diagnostic assays, such as HIV DNA or RNA PCR, are considerably more useful in young infants, allowing a definitive diagnosis in most infected infants by 1-4mo of age. By 4mo of age, HIV PCR testing identifies all infected nonbreastfed infants.

Almost 40% of infected newborns have positive test results in the first 2 days of life, with >90% testing positive by 2 wk of age. Either the DNA or RNA PCR is considered acceptable for infant testing.

Nelson 21st p1787 Table 302.3 Laboratory diagnosis of HIV infection

HIV DNA PCR

  • historically preferred to diagnosis HIV-1 subtype B infection in infants and children younger than 24mo of age
  • highly sensitive and specific by 2wk of age, available
  • performed on peripheral blood mononuclear cells
  • false negatives can theoretically occur in non-B subtype HIV-1 infections
  • historically had been preferred for testing in young infants

HIV RNA PCR

  • preferred test to identify non-B subtype HIV-1 infections
  • similar sensitivity and specificity to HIV DNA PCR in infants and children younger than 24mo of age “
How well did you know this?
1
Not at all
2
3
4
5
Perfectly
61
Q

“Treatment for uncomplicated gonococcal infections

a. Penicillin
b. Cefuroxime
c. Ceftriaxone
d. Clindamycin”

A

C

“Nelson 21st p1482 Table 219.1 Recommended treatment of gonococcal infections

Neonates

  • ophthalmia neonatoruum: ceftriaxone SD + lavage infected eye frequently until discharge eliminated
  • all other infections: ceftriaxone or ceoftaxime

Children <=45kg

  • all infections: ceftriaxone
  • pharyngeal, anorectal, urogenital infections, conjunctivitis: single dose

Adults, adolescents, and children >45kg

  • all infections: ceftriaxone + azithromycin SD
  • pharyngeal, anorectal, urogenital infection, conjunctivitis: single dose ceftriaxone + single dose azithromycin
  • Disseminated infection, septic arthritis, scalp abscess - 7 days duration
  • Meningitis - 10-14 days duration
  • Endocarditis - minimum 28 days duration”
62
Q
"Vaccines given by DOH last September 2014 due to outbreak
A. Varicella
B. Measles
C. Measles and Rubella 
D. Flu"
A

C

63
Q
"A congenital infection associated with non-communicating or communicating hydrocephalus 
A. Toxoplasmosis
B. Rubella
C. TB
D. HIV"
A

A

“Nelson 21st p1871
Neurologic manifestations of congenital toxoplasmosis vary from massive acute encephalopathy to subtle neurologic syndromes.

Hydrocephalus may be the sole clinical manifestation of congenital toxoplasmosis and almost always requires shunt placement. Hydrocephalus may be present prenatally and progress during the perinatal period or, much less often, may present later in life.

Calcifications occur throughout the brain, but there is propensity for development of calcfications in the caudate nucleus and basal ganglia, choroid plexus, and subependyma. “

64
Q
"The only deep-seated fungal infection with no cutaneous manifestation.
A. Candida
B. Aspergillus 
C. Pityriasis versicolor 
D. Blastomycosis"
A

NOTA

“Nelson 21st p1653 Blastomycosis
The most common clinical manifestation of blastomycosis is pneumonia, which can range from acute to chronic. Extrapulmonary blastomycosis most often affects the skin or bone but can involve almost any organ

Nelson 21st p1648 Aspergillosis
The most common site of primary infection is the lung, but primary invasive infection is also seen in the sinuses and skin and rarely elsewhere. Secondary infection can be seen after hematogenous spread, often to the skin, CNS, eye, bone, and heart. “

65
Q
"Patient with staphylococcal infection allergic to penicillin or cephalosphorins. Staph is penicillin-resistant but methicillin-sensitive. What will you give?
A. Meropenem
B. Vancomycin
C. Clindamycin
D. Linezolid"
A

B

“Nelson 21st p1432 Table 208.1 Parenteral antimicrobial agents for treatment of serious staphylococcal infections.

Initial empiric therapy (organism of unknown susceptibility)

  1. Vancomycin + nafcillin or oxacillin - for life threatening infections (e.g. septicemia, endocarditis, CNSI); linezolid can be substituted if patient has previously recieved vancomycin
  2. Vancomycin - for non-life threatening infection without signs of severe sepsis (e.g. skin infection, cellulitis, osteomyelitis) with high rate of community acquired MRSA
  3. Cefazolin or nafcillin - for non-life threatening infection with low rate of community acquired MRSA
  4. Clindamycin - for non-life threatening infection with low rate of community acquired MRSA and low resistance to clindamycin

MSSA, penicillin resistant

  1. Nafcillin, cefazolin
  2. Clindamycin - for patients with serious penicillin-allergy and clindamycin-susceptible strain
  3. Vancomycin - only for penicillin and cephalosporin allergic patients
  4. Ampicillin-sulbactam - broader coverage including gram negatives

MRSA

  1. Vancomycin, clindamcyin
  2. Daptomycin, linezolid
  3. TMP-SMX”
66
Q

“Fungal disease predominant in HIV.

a. Cryptococcus
b. Histoplasma
c. Aspergillus
d. Toxoplasma”

A

A

67
Q
"Case of a child with fever, petechiae and rashes on hands and feet, with painful inguinal LAD:
A. Dengue hemorrhagic fever
B. Rocky mountain spotted fever
C. Rat bite fever
D. Leptospirosis"
A

B

“Nelson 21st p1624 RMSF (Rickettsia rickettsii)
In 49% of cases, patients or their parents report a history of removing an attached tick, although the site of the tick bite is usually inapparent. Epidemiologic clues involve living in/visiting an edemic area, playing or hiking in the woods, similar illness in family members, and close contact with a dog.

Manifestations often include fever, rash (frequently involving palms and soles), nausea and vomiting, headache. Less often with myalgias, abdominal pain, diarrhea, conjunctival injection, altered mental status, lymphadenopathy, and peripheral edema. Pain and tenderness of calf muscles are particularly common in children.

Nelson 21st p1602 Leptospirosis
The septicemic phase of anicteric leptospirosis has an abrupt onset with flulike signs of fever, shaking chills, lethargy, severe headache, malaise, nausea, vomiting, and severe debilitating myalgia most prominent in the lower extremities, lumbosacral spine, and abdomen.

Conjunctival suffusion with photophobia and orbital pain (in the absence of chemosis and purulent exudate), generalized lymphadenopathy, and hepatosplenomegaly may also be present. A transient erythematous maculopapular, urticarial, petechieal, purpuric, or desquamating rash occurs in 10% of cases.

Nelson 21st p3820 Rat bite fever (Streptobacillus moniliformis, Spirillum minus)

Streptobacillius: The illness is characterized by an abrupt onset of fever up to 41C, severe throbbing headache, intense myalgia, chills, and vomiting. Shortly after the onset of the fever, a polymorphous rash occurs. In most patients, the rash consists of blotchy red maculopapular lesions that often have a petechial component; the distribution of the rash is vairable, but is typically most dense on the extremities. Hemorrhagic vesicles may develop on the hands and feet and are very tender on palpation. Approximately 50% of patients have arthritis.

Spirillum: The hallmark of Spirillum-induced disease is fever associated with an indurated, often suppurative, nonhealing lesion at the bite site. Lymphadenitis and lymphadenopathy are invariably present in the regional nodes that drain the inoculation site, and many patients have a generalized macular rash most prominent when fever is present.

Nelson 21st p1762 Dengue
A majority of infected older children and adults experience sudden onset of fever, usually accompanied by frontal or retroorbital pain. Occasionally, severe back pain precedes the fever. A transient, macular, generalized rash that blances under pressure may be seen during the first 24-48hr of fever. Myalgia and arthralgia occur soon after the onset of fevers and increase in severity over time. From the second to sixth day of fever, nausea and vomiting are apt to occur, and generalized lymphadenopathy, cutaneous hyperesthesia or hyperalgesia, taste abberations, and pronounced anorexia may develop.

Approximately 1-2 days after defervescence, a generalized, morbilliform, maculopapular rash appears that spares the palms and soles. It disappears in 1-5 days, and desquamation may occur. “

68
Q
"Child with watery, mucoid stools with streaks of blood, associated with abdominal pain, tenesmus, and sense of defecation. What is the most common etiologic agent?
A. Salmonella
B. Shigella
C. ETEC
D. Cholera"
A

B

“Nelson 21st p2020
Global Enteric Multicenter study: most episodes of moderate-to-severe diarrhea were attributed to 4 pathogens: rotavirus, Cryptosporidium, Shigella, and ETEC.

The 3 agents associated with the most deaths among children under 5yr are rotavirus, Cryptosporidium, and Shigella

Etiology, Risk Factors, and Interactions of Enteric Infections and Malnutrition and the Consequences for Child Health and Development project: study of less severe, community based diarrhea - viral cause predominated, but Shigella had the single highest atrributable incidence. “

69
Q

“Initially started on GI tract and later affects the CNS

a. Rabies
b. Polio
c. Tetanus
d. Jap enceph”

A

B

“Nelson p1683
In the contact host, wild-type and vaccine strains of poliovirus gain host entry via the gastrointestinal tract. Recent studies in nonhuman primates demonstrate that the primary sites of replication are in the CD155 epithelial cells lining the mucosa of the tonsil follicle and small intestine, as well as in the macrophages/dendritic cells in the tonsil follicle and Peyer patches. Regional lymph nodes are affected, and primary viremia occurs after 2-3 days. The virus seeds multiple sites, including the reticuloendothelial system, brown fat deposits, and skeletal muscle. Wild type polioviris probably accesses the CNS along peripheral nerves. “

70
Q

“A case of a prepubertal female with vaginal discharge described as white and cheesy. What will you do?
A. Observe
B. Start systemic antibiotics
C. Refer to gynecology
D. Candida infection is considered as it is commonly seen among this age”

A

C

“Nelson 21st p1642
Vulvovaginitis is a common Candida infection of pubertal and postpubertal female patients. Prepubertal girls with Candida vulvovaginitis usually have a predisposing factor such as DM or prolonged antibiotic treatment.

Clinical manifestations include pain or itching, dysuria, vulvar or vaginal erythema, and an opaque white or cheesy exudate. Candida vulvovaginitis can be effectively trated with either vaginal creams or troches of nystatin, clotrimazole, or miconazole. Oral therapy with a single dose of fluconazole is also effective.

Nelson 21st p109
A number of STIs should raise concern for abuse. In a prepubertal child, gonorrhea or syphilis beyond the neonatal period indicates that the child has had some contact with infected genital secretions, almost always as a result of sexual abuse. There is some evidence that chlamydia in children up to 3 yr of age may be perinatally acquired. Chlamydia in children >3 yr old is diagnostic of contact with infected genital secretions, almost always as a result of sexual abuse. HIV is diagnostic for sexual abuse if other means of transmission have been excluded. Because of the potential for transmission either perinatally or through nonsexual contact, the presence of genital warts has a low specificity for sexual abuse. The possibilty of sexual abuse should be considered and addressed, especially in children whose warts first appear beyond 5yr of age. “

71
Q

“25 day old with pertussis, what is the treatment of choice?

a. Erythromycin
b. Azithromycin
c. Clarithromycin
d. Gentamicin”

A

B

“Nelson 21st p1494
Azithromycin is the drug of choice in all age groups, for treatment or post eposure prophylaxis. TMP-SMX is an alternative to azithromycin for infants >2mo old and children unable to recieve azithromycin.

Infantile hypertrophic pyloric stenosis is associated with macrolide use in young infants, especially in those <14 days old, with highest risk in those recieving erythromycin. “

72
Q

“Most common cause of wheezing in infant -

a. Bronchiolitis
b. GERD
c. Bronchitis
d. Asthma”

A

A

73
Q

“15 month old from Tondo with infiltrates on CXR, no retractions, wbc 20,000

a. Pneumonia, severe
b. Pneumonia, non-severe
c. Bronchiolitis
d. Tracheitis”

A

B

“Nelson 21st p2219
The diagnosis of acute bronchiolitis is clinical, particularly in a previously healthy infant presenting with a first episode of wheezing following a period of upper respiratory symptoms. Chest radiography is is not routinely indicated in children with suspected bronchiolitis. Areas of atelectasis associated with bronchiectasis are often observed on CXR and may be difficult to distinguish from bacterial pneumonia. Laboratory testing is also not routinely indicated; the WBC and differential counts are usually normal and not predictive of bacterial superinfection.

PAPP 2021 PCAP CPG p15-16
Criteria for severe pneumonia (requiring admission)

  1. Respiratory signs - cyanosis/hypoxemia, head bobbing, retractions, apena, grunting
  2. CNS signs - lethargic/stuporous/comatose/GCS <13, seizures
  3. Circulatory signs - capillary refill >3s or shock, pallor
  4. General considerations - moderate to severe malnutrition, refusal or inability to take food/drink per orem, some to severe dehydration, age <6 months
  5. Ancillary parameters - CXR or UTZ findings of consolidation, multifocal disease, moderate to large effusion, abscess, air leak; sustained O2 sat at room air <=93% “
74
Q

“Diarrhea with malabsorption, bloating

a. Giardia
b. E. coli
c. Enterococcus
d. Cholera”

A

A

“Nelson 21st p1834 Giardia
Most symptomatic patients usually have a limited period of acute diarrheal disease with or without low grade fever, nausea, and anorexia. In an small proportion of patients, an intermittent or more protracted course characterized by diarrhea, abdominal distension and cramps, bloating, malaise, flatulence, nausea, anorexia, and weight loss occurs.

Stools may initially be profuse and watery and later become foul smelling and may flloat. Stools do not contain blood, mucus, or fecal leukocytes. Varying degrees of malabsorption may occur. “

75
Q

“Amebic abscess presents with fever and _

a. Chills
b. Abdominal pain
c. Dysuria
d. Constipation”

A

B

“Nelson 21st p1832
Amebic liver abscess, a serious manifestation of disseminated infection, is uncommon in children. Fever is the hallmark of amebic liver abscess and is frequently associated with abdominal pain, distension, and enlargement and tenderness of the liver. Changes at the base of the right lung, such as elevation of the diaphragm and atelectasis or effusion, may also occur. “

76
Q

“Presents with lymphadenopathy other than TB

a. Measles
b. SARS
c. Atypical pneumonia
d. None of the above”

A

A

“Nelson 21st p1671-1672
Measles is a serious infection characterized by high fever, an enanthem, cough, coryza, conjunctivitis, and a prominent exanthem.

After an incubation period of 8-12 days, the prodromal phase begins with a mild fever followed by onset of conjunctivitis with photophobia, coryza, a prominent cough, and increasing fever.

Koplik spots represent the enanthem and are the pathognomonic sign of measles, appearing 1-4 days prior to the onset of the rash. They first appear as discrete red lesions with bluish white spots on the center on the inner aspects of the cheeks at the level of the premolars.

Symptoms increase in intensity for 2-4 days until the 1st day of the rash. The rash begins on the forehead, behind the ears, and on the upper neck as a red maculopapular eruption. It then spreads downwards to the torso and extremities, reaching the palms and soles in 50% of cases. The exanthem frequently becomes confluent on the face and upper trunk.

With the onset of the rash, symptoms begin to subside. The rash fades over about 7 days in the same progression as it eveolved, often leaving a fine desquamation of skin in its wake. Of the major symptoms, the cough last the longest, often up to 10 days.

In more severe cases, generalized lymphadenopathy may be present, with cervical and occipital lymph nodes especially prominent. “

77
Q

“2 year old child with sore throat, inflamed gums, ulcers on tongue, drooling

a. Herpeticogingivostomatitis
b. Diphtheria
c. Epiglottitis
d. Kawasaki disease”

A

A

“Nelson 21st p1703
Herpes gingivostomatitis most often affects children 6mo to 5yr of age but is seen across the age spectrum. It is an extremely painful condition with sudden onset, pain in the mouth, drooling, refusal to eat or drink, and fever up to 40-40.6C.

The gums become markedly swollen, and vesicles may develop throughout the oral cavity, including the gums, lips, tongue, palate, tonsils, pharynx, and perioral skin. The vesicles are generally present only a few days before progressing to form shallow indurated ulcers that may be covered by a yellow-gray membrane

Tender submandibular, submaxillary, and cervical lymphadenopathy is common. The breath may be foul as a result of overgrowth of anaerobic oral bacteria.”

78
Q

“Which specimen will help diagnose CMV in neonates?

a. Urine
b. Blood
c. CSF
d. None of the above”

A

A

“Nelson 21st p1722
The diagnosis of congenital CMV infections requires the recovery of replicating virus and/or viral nucleic acids within the first 2-3 wk of life. Sources of virus and viral nucleic acids include urine, saliva and blood.

Studies have indicated that the sensitivity of dried blood spots is too low to be considered useful for screening. Newborn screening using saliva has proven sensitive and specific and is now standard for newborn screening in some institutions. Identification of an infected infant by screening of saliva requires confirmation, preferably by assaying urine for the presence of CMV. “

79
Q

“Your 2 year old patient was exposed to his brother who has varicella. What will you do?

a. give Varicella vaccine
b. give human varicella zoster immunoglobulin
c. give Acyclovir
d. Give varicella vaccine and acyclovir”

A

A

“Nelson 21st p1715
Vaccine given to healthy children within 3-5 days after exposure (as soon as possible is preferred) is effective in preventing or modifying varicella. Varicella vaccine is now recommended for postexposure use and for outbreak control.

High titer anti-VZV immune globulin as postexposure prophylaxis is recommended for immunocompromised children, pregnant women, and newborns exposed to varicella: Newborns whose mothers have varicella 5 days before to 2 days after delivery should recieve VariZIG. VariZIG is also indicated for pregnant women and immunocompromised persons without evidence of varicella immunity. “

80
Q

“A 4 year old girl was bitten by a rat at her forefinger, which is spontaneously bleeding when you examined the patient. What is your management?

a. Give antirabies vaccine
b. Give Penicillin
c. Give azithromycin
d. None of the above”

A

B

“Nelson 21st p3819
Amoxicillin-clavulanate is an excellent choice for empirical oral therapy for human and animal bite wounds because of its activity against most bacteria that have been isolated from infected bites. Similarly, piperacillin-tazobactam or ampicillin-sulbactam is preferred for patients who require empirical parenteral therapy.

Penicillin G remains the drug of choice for prophylaxis and treatment of rat-inflicted injuries, as this agent has excellent activity against Streptobacillus and Spirillum

Although tetanus occurs only rarely after human or animal bite injuries, it is important to obtain a careful immunization history and to provide tetatnus toxoid to all patients who are incompletely immunized or who have gone longer than 5 years since their last tetanus immunization.

Nelson 21st p1774-1775
Theoretically, rabies virus can infect any mammal, but true animal reservoirs that maintain the presence of rabies virus in the population are limited to terrestrial carnivores and bats. Rabies is rare in small mammals, including mice, squirrels, and rabbits. To date no animal-to-human transmission from these animals has been documented. “

81
Q

“A 7 year old was bitten by a cat at his left foot. The boy was given pre exposure prophylaxis before. What will you give the patient ?

a. Give PCEC/PVRV at day 0, 7, 21, 28
b. Give HRIG
c. Give booster dose at day 0 and day 3
d. None of the above”

A

C

“Nelson 21st p1776-1777
Post exposure prophylaxis
The first step in rabies PEP is to cleanse the wound thoroughly. Soapy water is sufficient to inactivate an eveloped virus. Other commonly used disinfectants, such as iodine-containing preperations, are virucidal and should be used in addition to soap if possible.

The second component of rabies PEP consists of passive immunization with RIG. Human RIG is administered at a dose of 20IU/kg. As much of the dose is infused around the wound as possible, and the remainder is injected IM in a limt distant from the one injected with the killed vaccine.

The third component of rabied PEP is immunization with inactivated virus. In both children and adults, both vaccines are administered IM in a 1ml volume in the deltoid and anterolateral thigh on days 0,3,7, and 14 after presentation.

Preexposure prophylaxis
Preexposure prophylaxis should be considered for persons traveling to a rabies-endemic region where there is a credible risk for a bite or scratch from a rabies-infected animal.

The schedule for preexposure prophylaxis consists of 3 IM injections on days 0,7, and 21 or 28.

Post exposure prophylaxis who has recieved preexposure prophylaxis or a prior full schedule of post exposure prophylaxis consists of 2 doses of vaccine (one each on days 0 and 3) and does not require RIG

Immunity from preexposure prophylaxis and requires boosting if the potential for exposure to rabid animals occurs. “

82
Q

“11 year old unimmunized. What to give?

a. 3 doses of DPT
b. 1 dose of DTaP and 2 doses of Td
c. 3 doses of DTaP
d. 4 doses of TDAP”

A

B

“Prev Ped 2018 p74 Summary table 2018: Immunization of preadolescents and adolescents (7-18 years old)

Hep B vaccine
- unvaccinated 7-18 yrs old, 3 doses needed at 0-1-6 months

Hep A vaccine

  • unvaccinated 7-18 yrs old, 2 doses needed
  • 2nd dose given at least 6 months from the 1st dose

MMR

  • unvaccinated 7-18 yrs old: 2 doses needed
  • incompletely vaccinated 7-18 yrs old: 1 dose needed
  • 4 weeks interval between doses

Varicella

  • unvaccinated 7-12 yrs old: 2 doses needed, 3 months interval
  • unvaccinated >=13 yrs old: 2 doses needed, 1 month interval
  • incompletely vaccinated 7-18 yrs old - 1 dose, same intervals as above

Influenza vaccine
- 9-18 yrs old - 1 dose, give annnually beginning Feb

Td/Tdap

  • unvaccinated 7-18 yrs old: 3 doses needed, 0-1-6 months
  • incompletely vaccinated 7-18 yrs old: 1-2 doses needed, 1 dose Tdap then Td for the remaining dose
  • Fully vaccinated 7-18yrs old - 1 dose Tdap then Td every 10 years

HPV

  • Bivalent - females 15-18 yrs, 3 doses 0-1-6 months
  • Quadrivalent - males and females 15-18yrs, 3 doses 0-2-6 months “
83
Q

“Etiologic agent that causes slapped cheek appearance, rash that is lacy, reticulated appearance

a. HSV
b. Enterovirus
c. ParvovirusB19
d. Adenovirus”

A

C

“Nelson 21st p1699
The most common manifestation of Parvovirus B19 is erythema infectiosum, also known as fifth disease, which is a benign, self-limited exanthematous illness of childhood.

The hallmark of erythema infectiosum is the characteristic rash, which occurs in 3 stages that are not alwys distinguishable. The initial stage is an erythematous facial flushing, often described as a slapped-cheek appearance. The rash spreads rapidly or concurrently to the trunk and proximal extremities as a diffuse macular erythema in the second stage. Central clearing of macular lesions occurs promptly, giving the rash a lacy, reticulated appearance.

Nelson 21st p1692-1694
Clinical manifestations of enteroviruses
1. Hand-foot-and-mouth disease (coxackievirus A16, enterovirus A71) - scattered painful vesicles on hand, feet, buttocks, groin, oral cavity
2. Herpangina (enterovirus A71) - sudden onset of fever, sore throat, dysphagia, painful lesions in the posterior pharynx
3. Pleurodyna/Bornholm disease (coxsackie B virus) - paroxysmal thoracic pain due to myositis involving chest and abdominal walla muscles and possibly pleural inflammation
4. Acute hemorrhagic conjunctivitis (enterovirus D70, coxsackievirus A24) - severe eye pain with photophobia, blurred vision, lacrimation, conjunctival erythema, subconjunctival hemorrhage
5. Myocarditis/pericarditis - enteroviruses account for approximately 25-35% of cases of myocarditis or pericarditis with proven etiology
6. Orchitis - coxsackie B viruses are second only to mumps as causes of orchitis
7. Viral meningitis - enteroviruses are the most common cause of viral meningitis in mumps-immunized populations “

84
Q

“17 year old boy works at a farm, where he takes care of the cows and sheep. He developed fever, chills, conjuctival suffusion and myalgia especially at the leg. What is your consideration?

a. Dengue
b. Chikungunya
c. Leptospirosis
d. Toxoplasmosis”

A

C

“Nelson 21st p1602 Leptospirosis
The septicemic phase of anicteric leptospirosis has an abrupt onset with flulike signs of fever, shaking chills, lethargy, severe headache, malaise, nausea, vomiting, and severe debilitating myalgia most prominent in the lower extremities, lumbosacral spine, and abdomen.

Conjunctival suffusion with photophobia and orbital pain (in the absence of chemosis and purulent exudate), generalized lymphadenopathy, and hepatosplenomegaly may also be present. A transient erythematous maculopapular, urticarial, petechieal, purpuric, or desquamating rash occurs in 10% of cases. “

85
Q

“11 year old girl with fever for 2 days, severe arthralgia, rash that blanches with pressure. Diagnosis?

a. Dengue
b. Chikungunya
c. Rubella
d. Malaria”

A

A

“Nelson 21st p1756 Chikungunya
Endemic in urban Aedes aegypti or Aedes albopictus transmission cycles in Africa and Asia.

The onset is abrupt, with high fever and often severe joint symptoms (hands, feet, ankles, wrists) that include symmeptric bilateral polyarthralgia or arthritis. There may be headache, myalgias, conjunctivitis, weakness, lymphopenia and maculopapular rash.

Nelson 21st p1762 Dengue
A majority of infected older children and adults experience sudden onset of fever, usually accompanied by frontal or retroorbital pain. Occasionally, severe back pain precedes the fever. A transient, macular, generalized rash that blances under pressure may be seen during the first 24-48hr of fever. Myalgia and arthralgia occur soon after the onset of fevers and increase in severity over time. From the second to sixth day of fever, nausea and vomiting are apt to occur, and generalized lymphadenopathy, cutaneous hyperesthesia or hyperalgesia, taste abberations, and pronounced anorexia may develop.

Approximately 1-2 days after defervescence, a generalized, morbilliform, maculopapular rash appears that spares the palms and soles. It disappears in 1-5 days, and desquamation may occur. “

86
Q
"What is the most common clinical manifestation of Neiserria meningitidis infection?
A. Fever
B. Meningitis
C. chronic meningococcemia
D. occult bacteremia"
A

B

“Nelson 21st p1471-1472
The most common form of meningococcal infection is asymptomatic carriage of the organism in the nasopharynx. In the rare cases where invasive diseasee occurs, the clinical spectrum of meningococcal disease varies widely, but the highest proportion of cases present with meningococcal meningitis (30-50%). Other recognized presentations include bacteremia without sepsis, meningococcal septicemia with or without meningitis, pneumonia, chronic meningococcemia, and occult bacteremia. “

87
Q

“What is the period of maximum infectiousness of Mumps?
A. 7 days before to as long as 7 days after onset of parotid swelling
B. 1- 2 days before to 5 days after parotid swelling
C. 3 days before to 6 days after parotid swelling
D. 5 days before to 2 days after parotid swelling”

A

B

“Nelsons 21st p1680
Mumps is spread from person to person by respiratory droplets. Virus appears in the saliva from up to 7 days before to as long as 7 days after onset of parotid swelling. The period of manixmum infectiousness is 1-2 days before to 5 days after onset of parotid swelling.”

88
Q

“A 5-year old male presented with high grade fever, swollen right eyelid, proptosis, limitation of movement of the eye, &
edema of the conjunctiva. The empiric treatment should include:
A. Ophthalmic steroids
B. Ocular antihistamines
C. Vancomycin & cefotaxime
D. Incision & drainage”

A

C

“Nelson 21st p3392 Orbital cellulitis
Inflammation of the tissues of the orbit, characterized by the triad of proptosis, painful limitation of movement of the eye, and potentially decreased visual acuity, is termed orbital cellulitis. Edema of the conjunctiva (chemosis) and inflammation and swelling of the eyelids may be seen.

Ortbital cellulitis must be recognized promptly and treated aggressively. Hospitalization and systemic antibiotic therapy are indicated.

Antimicrobial agents should begin with IV ampicillin-sulbactam OR IV clindamycin plus ceftriaxone, cefepine or cefotaxime. In cases where there is suspicion for intracranial extension, vancomycin plus cefotaxime/ceftriaxone plue metronidazole should be given. “

89
Q

“The current guidelines for the diagnosis of acute otitis media are more restrictive compared to the earlier recommendations. Which of the following features is NOT a diagnostic criterion?
A. Acute onset of symptoms
B. Presence of middle ear effusion
C. Signs of acute middle ear inflammation
D. Localized swelling of the ear canal”

A

D

“Nelson 21st p3421
A diagnosis of AOM according to the 2013 American Academy of Pediatrics guidelines should be made in children who present with:
1. Moderate to severe bulging of the TM or new onset otorrhea not caused by otitis externa
2. Mild bulging of the TM and recent (<48hr) onset of ear pain or intense TM erythema

A diagnosis of AOM should not be made in children without middle ear effusion.

Both AOM and OME are accompanied by physical signs of middle ear effusion, namely the presence of a least 2 of 3 TM abnormalities: white, yellow, amber, or blue discoloration; opacification other than that cause by scarring; decreased or absent mobility.

To support a diagnosis of AOM instead of OME in a child with MEE, distinct fullness or bulging of the TM may be present, with or without accompanying erythema, or, at a minimum, MEE should be accompanied by ear pain that appears clinically important.

In otitis media with effusion (OME), bulging of the TM is absent or slight or the membrane may be retracted; erythema is also absent or slight but may increase with crying or with superficial trauma to the external auditory canal incurred in clearing the canal of cerumen. “

90
Q
"A 1-year old male, with a VP shunt, was admitted because of high-grade fever of 3 days. At the ER, he presented with signs of meningeal irritation . What organism should be considered?
A. Neisseria meningitidis
B. Staphylococcus epidermidis
C. Group B Streptococcus
D. Hemophilus influenzae"
A

B

“Nelson 21st p1435-1436
There are approximately 30 species of coagulase-negative staphylococci (CONS) affecting or colonizing humans: S. epidermidis, or less often, S. hominis, S. hemolyticus, and others, are widely distributed on the skin and are significant causes of nosocomial or device-associated infection.

CONS, introduced at surgery, is the most common pathogen associated with CSF shunt meningitis. Most infections (70-80%) occur within 2 mo of the operation and maniest as signs of meningeal irritation, fever, increased ICP, or peritonitis from the intraabdominal position of the distal end of the shunt tubing. “

91
Q

“Which of the following clinical features differentiates Weil syndrome from anicteric leptospirosis?
A. Debilitating myalgia of the lower extremities
B. Conjunctival suffusion with photophobia
C. Renal failure
D. Meningitis”

A

C

“Nelson 21st p1602 Leptospirosis
The septicemic phase of anicteric leptospirosis has an abrupt onset with flulike signs of fever, shaking chills, lethargy, severe headache, malaise, nausea, vomiting, and severe debilitating myalgia most prominent in the lower extremities, lumbosacral spine, and abdomen.

Conjunctival suffusion with photophobia and orbital pain (in the absence of chemosis and purulent exudate), generalized lymphadenopathy, and hepatosplenomegaly may also be present. A transient erythematous maculopapular, urticarial, petechieal, purpuric, or desquamating rash occurs in 10% of cases.

The second or immune phase can follow a brief asymptomatic interlude and is characterized by recurrence of fever and aseptic meningitis.

Weil syndrome is a severe form of leptospirosis seen more commonly in adults (>30yr) than in children. The initial manifestations are similar to those described for anicteric leptospirosis. The immune phase, however, is characterized by jaundice, acute renal dysfunction, thrombocytopenia, and in fulminant cases, pulmonary hemorrhage and cardiovascular collapse. “

92
Q
"3-year old female had productive cough & sore throat of 4 days. This was followed after 48 hours by the appearance of diffuse, finely papular, erythematous eruptions producing a bright red discoloration of the elbows, axillae & groin. The rash blanches on pressure. The cheeks were similarly erythematous with pallor around the mouth . What is the most probable causative agent?
A. Coxsackie virus
B. Group A Streptococcus
C. Hemophilus influenzae
D. Moraxella catarrhalis"
A

B

“Nelson 21st p1441-1442
Scarlet fever is GAS pharyngitis associated with a characteristic rash, which is caused by an infection with pyrogenic exotoxin-producing GAS in individuals who do not have antitoxin antibodies.

The rash appears within 24-48 hr after onset of symptoms, although it may appear with the first signs of illness. It often begins around the neck and spreads over the trunk and extremities. The rash is a diffuse, finely papular, erythematous eruption producing bright red discoloration of the skin, which blanches on pressure. It is often accenuated in the creases of the elbows, axilla, and groin. The skin has a goose-pimple appearance and feels rough. The cheeks are often erythematous with pallor around the mouth.

After 3-4 days, the rash begins to fade and is followed by desquamation, initially on the face, progressing downward, often resembling a mild sunburn. Occasionally, sheetlike desquamation may occur around the free margins of the fingernails, palms, and soles.

Examination of the pharynx of a patient reveals essentially the same findings as with GAS pharyngitis. In addition, the tongue is usually coated and the papillae are swollen. After desquamation, the reddened papillae are prominent, giving the tongue a strawberry appearance. “

93
Q
"A 4-year old male presented with high grade fever, irritability, productive cough, refusal to drink, vomiting & limb pain of 2 days. After 2 days, his hands & feet were noted to be cold with prolonged capillary refill time & non-blanching petechial & purpuric rash. What empiric treatment should be started?
A. Penicillin G
B. Clarithromycin
C. Clindamycin
D. Ampicillin + amikacin"
A

A

“Nelson 21st p1474
Empirical antimicrobial therapy should be initiated immediately after the diagnosis of invasive meningococcal infection is suspected and cultures are obtained, using a third-generation cephalosporin to cover the most likely bacterial pathogens until the diagnosis is confirmed.

Once the diagnosis of B lactam sensitive meningococcal disease is confirmed in the laboratory, some authorities recommend a switch to penicillin.

Nelson 21st p1474 Table 218.2 Treatment of Neisseria meningitides invasive infections

  1. Penicillin G 300,000 ukday - does not clear carriage, and prophylaxis is required at the end of treatment
  2. Ampicillin 200-400 mkday - does not clear carriage, and prophylaxis is required at the end of treatment
  3. Cefotaxime 200-300 mkday - recommended in the neonate
  4. Ceftriaxone 100mkday - preferred treatment as only given BID to OD, may reduce skin complications

*Alternative therapy in patient’s with allergy: Chloramphenicol, meropenem”

94
Q

“A 3-year old female had bloody diarrhea associated with severe abdominal pain, high grade fever, vomiting, urgency &
painful defecation. The stool examination showed> 50 PMN. What is the most probable diagnosis?
A. Bacillary dysentery
B. Campylobacter diarrhea
C. ETEC diarrhea
D. Salmonellosis”

A

A

“Nelson 21st p1510 Bacillary dysentery/shigella
Presumptive data supporting a diagnosis of bacillary dysentery include the finding of fecal leukocytes (usually >50 or 100 PMNs per HPF), fecal blood, and demonstration in peripheral blood of leukocytosis with a dramatic left shift. The total peripheral WBC count is usually 5000-15000 cells/uL, although leukopenia and leukemoid reactions occur.

Nelson 21st p1521 Campylobacter
The clinical presentation of Campylobacter enteritis can be similar to that of enteritis caused by other bacterial pathogens. Fecal leukocytes are found in as many as 75% of cases, and fecal blood is present in 50% of cases. Campylobacter should be considered in patients with bloody stools, fever, and abdominal pain.

Nelson 21st p1500 Salmonellosis
The most common clinical presentation of salmonellosis is acute enteritis. The stool typically contains a moderate number of PMNs and occult blood. Mild leukocytosis may be detected.

Nelson 21st p1515 E. coli
Other laboratory data are at best nonspecific indicators of etiology. Fecal leukocyte examination of the stool is often positive with EIEC or occasionally positive with other diarrheagenic E. coli.

Nelson 21st p1512 ETEC
The typical signs and symptoms include explosive watery nonmucoid nonbloody diarrhea, abdominal pain, nausea, vomiting, and little or no fever. “

95
Q
"An 18-month old male was admitted due to fever & throat pain. On examination, he was weak-looking & irritable. The tonsils were enlarged & covered with greyish exudates. He had enlarged & tender anterior & posterior cervical nodes, bull-neck appearance & inspiratory strider . What is the most probable diagnosis?
A. Peri-tonsillar abscess
B. Diphtheria
C. Retropharyngeal abscess
D. Epiglottitis"
A

B

“Nelson 21st p1459
In tonsillar and pharyngeal diphtheria, sore throat is the universal early symptom. Only half of patients have fever, and fewer with dysphagia, hoarseness, malaise, or headache. Mild pharyngeal injection is followed by unilateral or bilateral tonsillar membrane formation, which can extend to involve the uvula, soft palate, posterior oropharynx, hypopharynx, or glottic areas. Underlying soft tissue edema and enlarged lymph nodes can cause a bull neck appearance. The degree of local extension correlated directly with profound prostration, bull neck appearance, and fatality due to airway compromise or toxin-mediated complications.

Within a few days of respiratory tract infection (usually in the pharynx), a dense necrotic coagulum of organisms, epithelial cells, fibrin, leukocytes, and erythrocytes forms, initially white and advancing to become a gray-brown, leather like adherent pseudomembrane. Removal is difficult and reveals a bleeding edematous submucosa. “

96
Q

“Which of the following statements is TRUE of neonatal tetanus?
A. It typically manifests within 3-12 days of birth.
B. The incidence is higher among preterm neonates.
C. The spasms are usually cephalic.
D. The initial presentation in the majority of cases is trismus.”

A

A

“Nelson 21st p1550
Neonatal tetanus, the infantile form of generalized tetanus, typically manifests within 3-12 days of birth. It presents as progressive difficulty in feeding (sucking and swallowing), associated hunger, and crying. Paralysis and diminished movement, stiffness and rigidity to the touch, and spasms, with or without ophistotonos, are characteristic. The umbilical stump, which is typically the portal of entry for the microorganism, may retain remnants of dirt, dung, clotted blood, or serum, or may appear relatively benign. “

97
Q
"A 5-year old female was admitted because of on & off fever of 7 days. She had diarrhea for the past 3 days & complained of generalized myalgia & anorexia . On examination, she had maculo-papular rash appearing in crops on the lower chest, abdominal tenderness & hepatosplenomegaly. What is the most probable diagnosis?
A. Measles
B. Dengue fever
C. Typhoid fever
D. Infectious mononucleosis"
A

C

“Nelson 21st p1504
Typhoid fever usually manifests as high grade fever with a wide variety of associated features, such as generalized myalgia, abdominal pain, hepatosplenomegaly, and anorexia. In children, it may occur in the earlier stages of the illness and may be followed by constipation.

In approximately 25% of cases, a macular or maculopapular rash (rose spots) may be visible around 7th-10th day of the illness, and lesions may appear in crops of 10-15 on the lower chest and abdomen and last 2-3 days.

Patients managed as outpatients present with fever, but have less emesis, diarrhea, hepatomegaly, splenomegaly, and myalgias than patients who require hospital admission. “

98
Q
"A 2-year old male was admitted with a 3-day history of high grade fever & rash. There was marked tenderness of the skin. On the 2ND hospital day, crusting & fissuring around the eyes & mouth were noted. Desquamation occurred with gentle pressure on the skin. What is the most probable diagnosis?
A. Steven-Johnson syndrome
B. Measles
C. Erythema multiforme
D. Staphylococcal scalded skin syndrome"
A

D

“Nelson 21st p3553
Staphylococcal scalded skin syndrome, which occurs predominantly in infants and children younger than 5yr of age, includes a range of disease from localized bullous impetigo to generalized cutaneous involvement with systemic illness.

Onset of the rash may be preceded by malaise, fever, irritability, and exquisite tenderness of the skin. Scarlatiniform erythema develops diffusely and is accentuated in flexural and periorificial areas. The conjunctivae are inflamed and occasionally become purulent. The brightly erythematous skin may rapidly acquire a wrinkled appearance, and in severe cases, sterile, flaccid blisters and erosions develop diffusely. Circumoral erythema is characteristically prominent, as is radial crusting and fussuring around the eyes, mouth, and nose. At this stage, areas of epidermis may separate in response to gentle shear force (Nikolsky sign). As large sheets of epidermis peel away, moist, glistening, denuded areas become apparent, initially in the flexures and subsequently over much of the body surface. “

99
Q

“A previously unimmunized boy was given Hepatitis B immunoglobulin as prophylaxis for a needle-stick exposure. If he were to receive measles vaccination, the recommended interval is:
A. None - it can be given on the same day
B. 1 week
C. 3 months
D. 1 year”

A

C

”"”Nelson 21st p1673 Table 273.3 Suggested intervals between Ig administration and measles vaccination

3 months - tetanus Ig, Hep A Ig, Hep B Ig
4 months - rabies Ig
5 months - varicella Ig
6 months - measles Ig in immunocompetent, CMV Ig
8 months - measles Ig in immunosuppressed, IVIG for immmune deficiencies, IVIG for ITP (400mg/kg)
10 months - IVIG for ITP (1000mg/kg)
11 months - IVIG for Kawasaki

Blood transfusion
0 months - washed RBCs
3 months - RBCs, adrenaline saline added
6 months - pRBC, whole blood
7 months - plasma or platelet products """
100
Q

“What is the indication for IV acyclovir in varicella-zoster infection?
A. Non-pregnant females> 12 years of age
B. Individuals > 12 months of age with chronic cutaneous disorder
C. Children receiving aerosolized corticosteroid therapy
D. Patients with concomitant thrombocytopenia & pneumonia”

A

C

“Nelson 21st p1713
Oral therapy wih acyclovir (20mg/kg/dose) given as 4 doses pe day for 5 days can be used to treat uncomplicated varicella in individuals at increased risk for moderate to severe varicella:
1. Nonpregnant individuals older than 12 yr of age
2. Individuals older than 12mo of age with chronic cutaneous or pulmonary disorders
3. Individuals recieving short term, intermittent, or aerosolized corticosteroid therapy
4. Individuals recieving long term salicylate therapy
5. Possible secondary cases among househould contacts.

To be most effective, treatment should be initiated as early as possible, preferably within 24hr of the onset of the exanthem. There is less clinical benefit if treatment is initiated more than 72hr after the onset of the exanthem. “

101
Q
"A 1-year old male, born to an HIV-positive mother, had recurrent respiratory symptoms. On examination, he had multiple lymphadenopathies & hepatosplenomegaly. His CD4 count was< 200 (10% of the total lymphocytes). He was started on anti-retroviral therapy. What is the most appropriate vaccine that he should be given?
A. DTaP, OPV
B. MMR
C. PCV, HiB
D. Varicella"
A

C

“Nelson 21st p1800-1801. Also see Fig 302.5
All HIV-exposed and HIV-infected children should recieve standard pediatric immunizations.

Live oral polio vaccine should not be given due to poor immunologic response in HIV+ children as well as concern for live vaccination in potentially immunocompromised children.

The risk and benefits of rotavirus vaccination should be considered in infants born to HIV-infected mothers.

In other situations, the considerable attenuation of the vaccine’s strains should be considered, and unless the infant has clinical symptoms of AIDS or a CD4 percentage of <15%, vaccination is likely appropriate.

Other live bacterial vaccines (e.g. BCG) should be avoided because of the high incidence of BCG related disease in HIV-infected infants. Varicella and MMR vaccines are recommended for children who are not severely immunosuppressed (CD4 >=15%, CD4 count >500), but these vaccines should not be given to severely immunocompromised children.

It is recommended that children with HIV recieve quadrivalent meningococcal conjugate vaccine at a younger age than the routine schedule. Adolescent vaccines are also important, including the Tdap booster and HPV vaccine. “

102
Q

“A 10-year old male was diagnosed with Dengue Shock Syndrome. Which of the following clinical features is expected?
A. Pulse pressure 10 mmHg
B. Irritability, mid -epigastric pain
C. Scattered petechiae on forehead & extremities
D. Hematocrit increased by 50”

A

A

“Nelson 21st p1764
Dengue shock syndrome is a medical emergency that may occur in any child who lives in or has a recent travel history to a tropical destination. Management begins with diagnostic suspicion and the understanding that shock often accompanies defervescence.

Management of dengue hemorrhagic fever and dengue shock syndrome includes immediate evaluation of vital signs and degrees of hemoconcentration, dehydration, and electrolyte imbalance. Close monitoring is essential for at least 48hr because shock may occur or recur precipitously, usually several days after the eonet of fever. Patients who are cyanotic or have labored breathing should be given oxygen.

Rapid intravenous replacement of fluids and electrolytes can frequently sustain patients until spontanous recovery occurs. Normal saline is more effective than Ringer lactated saline in treating shock. When the pulse pressure is <=10mmHg or when elevation of hematocrit persists after the replacement of fluids, Plasma or colloid preparations are indicated. Oral rehydration of children who are being monitored is useful.

Prophylactic platelet transfusions have no been shown to reduce the risk of hemorrhaging or improve low platelet counts. Use of vasopressors has not resulted in significant reduction of mortality rates opver that observed with simple supportive therapy. Corticosteroids do not shorten the duration of dease or improve the prognosis in children recieving careful supportive therapy. “

103
Q

“The management of rotavirus gastroenteritis should include:
A. An antiviral drug
B. Diluted milk formula for at least 24 hours & BRAT diet
C. Rotavirus vaccine especially among immunodeficient patients
D. Oral rehydration therapy & resumption of diet for age after rehydration”

A

D

“Nelson 21st p1746-1747
Avoiding and treating dehydration are the main goals of the treatment of viral enteritis. A secondary goal is maintenance of the nutritional status of the patient.

There is no routine role for antiviral drug treatment of viral gastroenteritis. Controlled studies show limited benefits for antidiarrheal drugs. Ig have been administered orally to both normal and immunodeficient patients with severe rotavirus gastroenteritis, but this treatment is currently considered experimental. Therapy with probiotic organisms such as Lactobacillus species has been shown to be helpful only in mild cases.

Rehydration via the oral route can be accomplished in most patients with mild to moderate dehydration. Severe dehydration requires immediated IV therapy followed by oral rehydration.

Modern oral rehydration solutions containing appropriate quantities of sodium and glucose promote the optimum absorption of fluid from the intestine. Other clear liquds, such as flat soda, fruit juice, and sports drinks, are inappropriate for the rehydration of young children with significant stool loss.

Prolonged (>12hr) administration of exclusive clear liquids or dilute formula is without clinical benefit and actually prolongs the duration of diarrhea. Breastfeeding should be continued during hydration.

Hypocaloric diets low in protein and fat such as BRAT (bananas, rice, cereal, applesauce, toast) have not been shown to be superior to a regular diet.

Vaccine-associated disease has been reported in vaccine recipients who have severe combined immunodeficiency disease (a contraindication) “

104
Q
"Rabies post -exposure prophylaxis is NOT indica ted in bites from:
A. Rats, mice, rabbits
B. Stray dog in residential area
C. Bats captured & held in laboratories
D. Escaped cat"
A

A

“Nelson 21st p1774-1775
Theoretically, rabies virus can infect any mammal, but true animal reservoirs that maintain the presence of rabies virus in the population are limited to terrestrial carnivores and bats. Rabies is rare in small mammals, including mice, squirrels, and rabbits. To date no animal-to-human transmission from these animals has been documented.

Worldwide, transmission from dogs accounts for >90% of cases. In Africa and Asia, other animals serrve as prominent reservoirs, such as jackals, mongooses, and raccoon dogs.

Among American domesticated pets, infected cats outnumber infected dogs probably because cats frequently prowl unsupervised and are not uniformly subject to vaccine laws. “

105
Q

“A 29-year old mother delivered a term baby boy via NSD. She claimed she had varicella on her 35TH week of pregnancy. What should you do?
A. Reassure her that the baby will be alright
B. Start parenteral acyclovir
C. Give varicella zoster immunoglobulin
D. Observe the baby in the hospital for the appearance of skin lesions”

A

A

“Nelson 21st p1710
Mortality is particularly high in neonates born to susceptible mothers who contract varicella around the time of delivery. Infants whose mothers demonstrate varicella in the period from 5 days prior to delivery to 2 days afterward are at high risk for severe varicella.

The infant’s rash usually occurs toward the end of the 1st week to the early part of the 2nd wk of life. Because the mother has not yet developed a significant antibody response, the infant recieves a large dose of virus without the moderating effect of maternal anti-VZV antibody.

If the mother demonstrates varicella more than 5 days prior to delivery, she may still pass virus to the neonate, but the infection is attenudated because of transmission of maternal anti VZV antibody across the placenta. This moderating effect of maternal antibody is present if delivery occurs after about 30wk of gestation, when maternal IgG is able to cross the placenta in significant amounts.

Because perinatally acquired varicella can be life-threatening, the infant should usually be treated with acyclovir when lesions develop.

Nelson 21st p1715
Newborns whose mothers have varicella 5 days before to 2 days after delivery should recieve VariZIG. VariZIG is also indicated for:
1. pregnant women and immunocompromised persons without evidence of varicella immunity
2. hospitalized premature infants born <28 wk of gestation or weight <1000g who are exposed to varicella regardless of maternal varicella immunity
3. hospitalized premature infants born at >=28 wk oof gesteation who were exposed to varicella and whose mothers have no evidence of varicella immunity”

106
Q
"Which of the following manifestations is expected in severe cases of malaria?
A. Anemia
B. Acidosis
C. Coma
D. Jaundice"
A

All are correct

“Nelson 21st p1854 Table 314.1 WHO criteria for severe malaria, 2000

  • impaired consciousness
  • prostration
  • respiratory distress
  • multiple seizures
  • jaundice
  • hemoglobinuria
  • abnormal bleeding
  • severe anemia
  • circuatory collapse
  • pulmonary edema

Nelson 21st p1854
P. falciparum is the most severe form of malaria and is associated with higher density parasitemia and a number of complications.

The most common complication is severe anemia, which also is associated with other malaria species. Serious complications that appear unique to P. falciparum include cerebral malaria, respiratory distress from metabolic acidosis, acute renal failure, hypotension, and bleeding diathesis. “

107
Q
"The treatment of congenital cutaneous candidiasis in preterm infants includes:
A. Systemic amphotericin
B. Oral fluconazole
C. Topical nystatin cream
D. Clotrimazole patch"
A

A

“Nelson 21st p1641-1642
Signs of invasive candidiasis in premature infants are often nonspecific and include temperature instability, lethargy, apena, hypotension, respiratory distress, abdominal distension and thrombocytopenia

In the absence of systemic manifestations, topical antifungal therapy is the treatment of choice for congenital cutaneous candidasis in full term infants. Congenital cutaneous candidiasis in preterm infants can progress to systemic disease, and therefore systemic therapy is warranted.

Amphotericin B deoxycholate has been the mainstay of therapy for systemic candidiasis and is active against both yeast and myelial forms. Nephrotoxicity, hypokalemia, and hypomagnesemia are common, but amphotericin B deoxycholate is better tolerated in infants that in adult patients.

Lipsosomal amphotericin is often associated with worse outcomes in infants and should only be used when urinary tract involvement can reliably be excluded. Fluconazole is often used instead of amphotericin B deoxycholate for treatment of invasive neonatal Candida infections because of its effectiveness and low incidence of side effects. It is particularly useful for urinary tract infections, obtaining high concentrations in urine. “

108
Q
"The Epstein - Barr virus was the first human virus to be associated with malignancy. Which of the following malignancies is associated with its proliferation?
A. Osteosarcoma
B. Teratoma
C. Nasopharyngeal cancer
D. Oral hairy leukoplakia"
A

C

“Nelson 21st p1718
Infection with EBV, the first human virus to ba associated with malignancy, accounts for up to 2% of cancers worldwide. Manipulation of infected cells by EBV to establish and maintain latency can lead to transformation and oncogenesis.

EBV is associated with lymphoid malignancies, such as Burkitt lymphoma, Hodgkin lymphoma, aggressive NK cell leukemia, T- and NK cell lymphoproliferative disorder, and epithelial cell malignancies such as nasopharyngeal carcinoma and gastric carcinoma. “

109
Q

“A 10 -month old male had moderate-grade fever, productive cough & conjunctivitis of 5 days. He was given
paracetamol as needed. On the day of consultation, fever lysed but maculo-papular rash were noted on the forehead & gradually progressed to the trunk. What is the most probable diagnosis?
A. Drug eruption
B. Measles
C. Roseola
D. Rube ll a”

A

B

“Nelson 21st p1671-1672
Measles is a serious infection characterized by high fever, an enanthem, cough, coryza, conjunctivitis, and a prominent exanthem.

After an incubation period of 8-12 days, the prodromal phase begins with a mild fever followed by onset of conjunctivitis with photophobia, coryza, a prominent cough, and increasing fever.

Koplik spots represent the enanthem and are the pathognomonic sign of measles, appearing 1-4 days prior to the onset of the rash. They first appear as discrete red lesions with bluish white spots on the center on the inner aspects of the cheeks at the level of the premolars.

Symptoms increase in intensity for 2-4 days until the 1st day of the rash. The rash begins on the forehead, behind the ears, and on the upper neck as a red maculopapular eruption. It then spreads downwards to the torso and extremities, reaching the palms and soles in 50% of cases. The exanthem frequently becomes confluent on the face and upper trunk.

With the onset of the rash, symptoms begin to subside. The rash fades over about 7 days in the same progression as it eveolved, often leaving a fine desquamation of skin in its wake. Of the major symptoms, the cough last the longest, often up to 10 days.

In more severe cases, generalized lymphadenopathy may be present, with cervical and occipital lymph nodes especially prominent.

110
Q

“A patient with a history of diarrhea presented with fever, renal dysfunction, thrombocytopenia and anemia. You would suspect:

a. HSP
b. HUS
c. TTP
d. None of the above”

A

B

“Nelson 21st p2732
Hemolytic-uremic syndrome is a common cause of community-acquired acute kidney injury in young children. It is the most common form of thrombotic microangiopathy in children. HUS is characterized by the triad of microangiopathic hemolytic anemia, thrombocytopenia, and renal insufficiency.

The most common form of HUS is caused by Shiga-toxin producing E. coli (STEC) which causes prodromal acute enteritis and is commmonly termed STEC-HUS or diarrhea-associated HUS. Several serotypes of E. coli can produce the toxin; O157:H7 is the most common in Europe and the Americas. “

111
Q

“RTA is characterized by:

a. Persistent pyuria
b. Persistent alkaline urine
c. Persistent acidic urine
d. None of the above”

A

D

“Nelson 21st p2761
Renal tubular acidosis is a disease state characterized by a non-anion gap (hyperchloremic) metabolic acidosis in the setting of a normal or near-normal GFR.

There are 4 main types. Proximal (Type II), classic distal (Type I), hyperkalemic (Type IV) and combined proximal and distal (Type III). Proximal RTA results from impaired bicarbonate reabsorption and distal RTA from failure to secrete acid.

Nelson 21st p2763 Proximal RTA
Urinalysis in patients with isolated pRTA is generally unremarkable. The urine pH is acidic (<5.5) because distal acidification mechanisms are intact in these patients.

Nelson 21st p2763 Distal RTA
Because of impaired hydrogen ion excretion, the urine pH cannot be reduced to <5.5 despite the presence of severe metabolic acidosis. Inability to secrete H+ is compensated by an increased K+ secretion distally, leading to hypokalemia.

Distal RTA shares features with pRTA, including non-anion gap metabolic acidosis and growth failure; distinguishing features of distal RTA inclue nephrocalcinosis and hypercalciuria.

Nelson 21st p2764 Hyperkalemic RTA
Laboratory tests reveal a hyperkalemic non-anion gap metabolic acidosis. Urine may be alkaline or acidic. Elevated urinary sodium levels with inappropriately low urine potassium levels reflect the absence of aldosterone effect.

Nelson 21st p2764
Once the presence of a non-anion gap metabolic acidosis is confirmed, the urine pH can help distinguish distal from proximal causes. A urine pH <5.5 in the presence of acidosis suggests pRTA, while patients with distal RTA typically have a urine pH >6.0”

112
Q

“Best to diagnose UTI in children

a. Urine culture 100,000 cfu/ml
b. Fever, flank pain, urine culture 10,000 cfu/ml
c. Fever and pyuria
d. Fever and leucocytosis”

A

A

“Nelson 21st p2789-2790
Two basic forms of UTIs are pyelonephritis and cystitis

Pyelonephritis is characterized by and or all of the following: abdominal, back, or flank pain, fever, malaise, nausea, vomiting, and occasionally, diarrhea. Fever may be the only manifestation; particular consideration should occur for a temperature >39C without another source lasting for more than 24hr for males and more han 48hr for females. Pyelonephritis is the most common serious bacterial infection in infants younger than 24mo of age who have fever without an obvious focus.

Cystitis indicates that there is only bladder involvement. Symptoms include dysuria, urgency, frequency, suprapubic pain, incontinence, and possibly malodorous urine. Cystitis does not cause high fever and does not result in renal injury.

Nelson 21st p2792
If the culture shows >50,000 colony forming units/ml of a single pathogen (suprapubic or catheter sample) and the urinalysis has pyuria or bacteriuria in a symptomatic child, the child is considered to have a UTI”

113
Q

“Adolescent with vitiligo, orthostatic hypotension. Labs: hypoNa, hyperK, hypogly and metabolic acidosis. Diagnosis.

a. Cushings
b. Addisons
c. Behcet
d. None of the above”

A

B

“Nelson 21st p2959 & 2963
Acquired primary adrenal insufficiency is termed Addison disease. The most common cause of Addison disease is autoimmune destruction of the glands. The glands may be so small that they are not visible at autopsy, and oly remnants of tissue are found in microscopic sections.

Nelson 21st p2965
In older children with Addison disease, symptoms include muscle weakness, malaise, anorexia, vomiting, weight loss, and orthostatic hypotension. These may be of insidious onset.

Such patients can present with acute decompensation (adrenal crisis) during relatively minor infectious illnesses. Some of these patients have been initially misdiagnosed with chronic fatigue syndrome, postmononucleosis syndrome, chronic Lyme disease, or psychiatric disorders.

Hyperpigmentation is often, but not necessarily present. Hyponatremia is present at diagnosis in almost 90% of patients. Hyperkalemia tends to occur later in the course of the disease in older children and is present in only half the patients at diagnosis. Normal K levels must never be presumed to rule out primary adrenal insufficiency.

Hypoglycemia and ketosis are common. Addison disease should be considered in any child with orthostatic hypotension, hyponatremia, hypoglycemia, and ketosis.

Nelson 21st p1289
Behcet disease is classified as a primary variable vessel vasculitis, emphasizing the invovlement of any size and type of blood vessel. BD is also recognized as an autoinflammatory disease.

Nelson 21st p1290 Table 186.1 Consensus classification of pediatric Behcet disease

  1. Recurrent oral aphthosis - at least 3 attacks per year
  2. Genital ulceration or aphthosis - typically with scarring
  3. Skin involvement - necrotic folliculitis, acneiform lesions, erythema nodosum
  4. Ocular involvement - anterior uveitis, posterior uveitis, retinal vasculitis
  5. Neurologic signs - meningoencephalitis, encephalomyelitis, pseudotumor cerebri, psychiatric signs
  6. Vascular signs - venous thrombosis, arterial thrombosis, arterial aneurysm

*The criteria requires the presence of oral ulcers and 2 other major features to diagnose Behcet disease”

114
Q
"18mos, first febrile UTI. What to request?
A. Renal utz
B. DMSA
C. VCUG
D. Xray"
A

A

”"”The goal of imaging studies in children with UTI is to identify anatomic abnormalities that predispose to infection, determine whether there is active renal involvement, and assess whether renal function is normal or at risk.

The AAP practice parameter recommends initial ultrasound of kidneys, ureters, and bladder for children 2-24 months old with a first episode of UTI

(Nelson 21st p2793-2794)”””

115
Q
"First symptom of VUR
A. Urinary incontinence
B. UTI
C. Renal failure
D. Oliguria"
A

B

”"”VUR is usually discovered during evaluation for UTI. Among these children, 80% are female, and the average age of diagnosis is 2-3 years.

(Nelson 21st p2797)”””

116
Q

An adolescent female came in due to sudden onset LLQ pain . What could be the cause?

A

KIDNEY STONES

“Nelson 21st p2836
Children with urolithiasis usually have gross of microscopic hematuria. If the calculus causes ureteral or pelvic obstruction, then severe flank pain (renal colic) or abdominal pain occurs. The calculus typically causes obstruction at areas of narrowing of the urinary tract - the ureteropelvic junction, and the ureterovesical junction. The ureter progressively narros distally, and the most narrow segment is the ureterovesical junction

The pain typically radiates anteriorly to the scrotum or labia. Often the pain is intermittent, corresponding to periods of obstruction of urine flow, which increases the pressure in the collecting system.

If the calculus is in the distal ureter, the child can have irritative symptoms of dysuria, urgency, and frequency. If the stone passes into the bladder, the child usually becomes asymptomatic. If the stone is in the urethera, dysuria and difficulty voiding can result, particularly in males. Some children pass small amounts of gravel-like material. “

117
Q

When to do surgical treatment for undescended testis

A

“Nelson 21st p2828
The congenital undescended testis should be treated surgically by 9-15mo of age. With anesthesia by a pediatric anesthesiologist, surgical correction at 6mo is appropriate, because spontaneous descent of the testis will not occur after 4mo of age. “

118
Q
"A patient, apparently well, developed sudden onset of hematuria. What could be the cause?
A. IgA nephropathy
B. Trauma
C. RPGN
D. PSGN"
A

A

“Nelson 21st p2720-2721
IgA nephropathy is the most common chronic glomerular disease in children. It is characterized by a predominance of Iga within mesangial glomerular deposits in the absence of systemic disease.

Gross hematuria often occurs within 1-2 days of onset of an upper respiratory or gastrointestinal infection, in contrast with the longer latency period observed in acute PSGN, and may ba ssociated with loin pain. Proteinuria is often <1000 mg/24hr in patients with asymptomatic microscopic hematuria. Normal serum levels of C3 in IgA nephropathy help distinguish this disorder from PSGN. Serum IgA level have no diagnostic value because they are elevated inoly 15% of pediatric patients. Diagnosis of IgA nephropathy requires a kidney biopsy. “

119
Q

“A patient came in due to fever with a history of skin lesions 3 weeks prior. PE showed hypertension, dry skin lesions.
What labs will you request ?
A. CBC, UA, ASO
B. CBC, UCS, serum electrolytes
C. CBC, UA, ASO, C3
D. CBC, UCS, BUN creatinine, serum electrolytes”

A

C

“Nelson 21st p2723-2724
Poststreptococcal GN is most common in children ages 5-12yr and uncommon before the age of 3 yr. The typical patient develops an acute nephritic syndrome 1-2 week after an antecedent streptococcal pharyngitis or 3-6 wk after a streptococcal pyoderma.

The acute phase generally resolves within 6-8wk. Although urinary protein excretion and hypertension usually normalize 4-6wk after onset, persistent microscopic hematuria can persist for 1-2 yr after the initial presentation.

Urinalysis demonstrates RBCs, often in association with RBC casts, proteinuria and PMNs. The serum C3 level is significantly reduced in >90% of patients in the acute phase, and returns to normal 6-8wk after the onset. A mild normochromic anemia may be present from hemodilution and low-grade hemolysis.

Confirmation of the diagnosis requires clear evidence of a prior streptococcal infection. The ASO titer is commonly elevated after a pharyngeal infection but rarely increases after streptococcal skin infections. The best single antibody titer to document cutaneous streptococcal infection is the anti-DNAse B level.

The clinical diagnosis of PSGN is quite likely in a child presenting with an acute nephritic syndrome, evidence of recent streptococcal infection, and low C3 level”

120
Q

“A 3 year-old female started on antibiotics for suspected UTI. Which of the following is true?
A. Positive urine culture collected from bag will confirm presence of UTI
B. Pyuria is both confirmatory and diagnostic
C. Negative urine culture rules out UTI hence discontinuation of antibiotics”

A

C

“Nelson 21st p2972
A urine culture is necessary for confirmation and appropriate therapy. According the the AAP 2011 CPG for children 2-24mo, in children who are not toilet-trained, a catheterized or suprapubic sample should be obtained. Alternatively the application of an adhesive, sealed sterile collection bag after disinfection of the skin of the genitals can be useful only if the urinalysis or culture is negative. The negative predictive value for urinalysis from a bag specimen is 99%. However, a positive culture can result from skin contamination, particularly in females and uncircumsiced males.

Nitrites and leukocyte esterase are often positive in infected urine. In febrile infants less than 60 days old, the presence of pyuria, nitrites, or leukocyte esterase has a high sensitivity and specificity for a UTI.

Microscopic hematuria is common in acute cystitis, but microhematuria alone does not suggest UTI. Pyuria suggests infection, but infection can occur in the absence of pyuria; this finding is more confirmatory than diagnostic. “

121
Q
"Advise to give for patients with nephrotic syndrome in remission EXCEPT
A. Update immunization/vaccines for age
B. Low salt, low fat diet
C. Limit fluid intake
D. Start antibiotic prophylaxis"
A

D

“Nelson 21st p2758-2759
Managing the clinical sequalae of nephrotic syndrome
1. Edema - In addition to sodium restriction (<1500mg daily), water/fluid restriction may be necessary if the child is hyponatremic. Diuresis may be augmented by the administration of loop diuretics (furosemide)
2. Dyslipidemia - should be managed with a low fat diet
3. Infections - Families of children with nephrotic syndrome should be counseled regarding the signs and symptoms of infections such as cellulitis, peritonitis, and bacteremia
4. Thromboembolism - Children who presnt with the clinical signs of thromboembolism should be evaluated by appropriate imaging studies to confirm the presence of a clot. Anticoagulation therapy in children with thrombotic events appears to be effective
5. Obesity and growth - Growth may be affeceted in children who require long-term corticosteroid therapy. Anticipatory dietary counseling is recommended.
6. Immunizations - Give full pneumococcal vaccine and influenza vaccination annually to the child and their household contacts. Defer vaccination with live vaccines until the prednisone dose is below either 1mg/kg daily or 2mg/kg on alternate days. Live vaccines are contraindicated in children recieving corticosteroid-sparing agents such as cyclophosphamide and cyclosporine “

122
Q

“A 10 year old boy initially presents with hematuria.Kidney biopsy done and was diagnosed to have lg A nephropathy. What treatment will you give?

a. ACE inhibitors
b. Corticosteroids
c. Omega 3 polyunsaturated fatty acids
d. None of the above”

A

A

“Nelson 21st p2721
Although IgA nephropathy does not lead to significant kidney damage in children, progressive disease develops in 20-30% of adult patients 15-20 yr after disease onset. Therefore, most children with IgA nehropathy do not display progressive renal dysfunction until adulthood, prompting the need for careful long-term followup.

The primary treatment of IgA nephropathy is appropriate blood pressure control and management of significant proteinuria. ACE-I and ARB are effective in reducing proteinuria and retarding the rate of disease progression. Fish oil, which cointains anti-inflammatory omega-3 polyunsaturated fatty acids, may decrease the rate of disease progression in adults. Corticosteroids reduce proteinuria and improve renal function in those patients with GFR >60ml/min/m2. It remains unclear if the effects of glucocorticoids deter progression to end-stage renal failure to a degree to offset their significant side effects. “

123
Q

“A kid in school had lid fluttering and head drop. Diagnosis?

a. Petit mal
b. Absence
c. Juvenile myoclonic epilepsy”

A

B

“Nelson 21st p3095
Absence seizures are generalized seizures consisting of staring, unresponsiveness, and eye flutter lasting usually for a few seconds.

Typical absences are associated with 3Hz spike-and-slow-wave discharges and with childhood absence epilepsy which has a good prognosis.

Atypical absences are associated with 1- to 2-Hz spike-and-slow-wave discharges, and with head atonia and myoclonus during the seizures. They occur in Lennox-Gastaut syndrome and similar syndromes, which have a poor prognosis. “

124
Q

“CSF picture of TB meningitis
a. Total WBC 500 with mostly lymphocytes, glucose 30 mg/ di, TP 5000
b. WBC 5O, glucose 25, TP 20
C. WBC 100, glucose 35, TP 100”

A

A

“TBIC 2016 p52
CSF may be clear, usually opalescent and can contain 50-500 white blod cells/mm3, with polymorphonuclear leukocytes predominant very early in the disease and with lymphocytes predominating later. CSF glucose level may be at the lower limits of normal early in the second stage and very low by the third stage. The protein content may be normal initially but rises to a very high concentration, at which pellicle forms on standing “

125
Q

“Indications for neuroimaging in a child with headache

a. (+) family history of migraine
b. Hypertension
c. Aura
d. No response to paracetamol”

A

None of the above

“Nelson 21st p3133. Table 613.7 Indications for Neuroimaging in Child with Headaches

  • Abnormal neurologic examination
  • Abnormal or focal neurologic signs or symptoms
    • Focal neurologic symptoms or signs developing during a headache (i.e., complicated migraine)
    • Focal neurologic symptoms or signs (except classic visual symptoms of migraine) develop during the aura, with fixed laterality; focal signs of the aura persisting or recurring in the headache phase
  • Seizures or very brief auras (<5 min)
  • Unusual headaches in children
    • Atypical auras, including basilar-type, hemiplegic
    • Trigeminal autonomic cephalalgia, including cluster headaches in child or adolescent
    • An acute secondary headache (i.e., headache with known underlying illness or insult)
  • Headache in children younger than 6 yr old or any child who cannot adequately describe his or her headache
  • Brief cough headache in a child or adolescent
  • Headache worst on first awakening or that awakens the child from sleep
  • Migrainous headache in the child with no family history of migraine or its equivalent”
126
Q

“When do you operate myelomeningocoele

a. Anytime
b. Within 3 days
c. Within 7 days
d. Whithin day of birth “

A

D

“Nelson 21st p3067
Surgery is often done within a day or so of birth but can be delayed for several days (except when there is a CSF leak) to allow the parents time to begin to adjust to the shock and to prepare for the multiple procedures and inevitable problems that lie ahead.”

127
Q
"Neurotransmitter not involved in anxiety?
a. Acetylcholine
b. Norepinephrine
C. Dopamine
d. Serotonin"
A

A

128
Q

“Most common cause of arterial ischemic stroke in children?

a. Complex congenital heart disease
b. Glomerulonephritis
c. Hypertension
d. None of the above”

A

A

“Nelson 21st p3209-3211
Strokes most often involve the middle cerebral aretery teritory but can occur in any cerebral artery of any size.

Three main categories of etiology should be considered: arteriopathy, cardiac disease, and hematologic disease

Arteriopathy, a disorder of the cerebral arteries, is a leading cause of childhood AIS, present in more than 50% of children. Examples include transient cerebral arteriopathy and moyamoya disease

Cardioembolic stroke makes up approximately 25% of childhood AIS cases, with the maximal embolic risk concurrent with catheterizetion, surgical repair, or ventricular assist device use. Although complex congenital heart diseases are most frequently associated with AIS, acquired conditions, including arrythmia, cardiomyopathy, and infective endocarditis, should be considered. A patent foramen ovale provides a possible conduit for paradoxical venous thromboembolism to the brain

Hematologic disorders associated with AIS include sickle cell anemia, in which the stoke risk is increased 400-fold. Coagulation disorders are associated with childhood AIS. They include hereditary and acquired prothrombotic states and prothrombotic medications, including oral contraceptives and asparaginase therapy “

129
Q

“A 4 year old went to ER after swallowing a battery. Noted in the esophagus. What should be done?
a. Observe with serial xray
b. Remove immediately because of risk of erosion
C. Endoscopy
d. Surgical referral”

A

B

“Nelson 21st p1943
Treatment of esophageal foreign bodies usually merits endoscopic visualization of the object and underlying mucosa and removal of the object using an appropriately designed foreign body-retrieving accessory instrument through the endoscope and with an endotracheal tube protecting the airway.

Sharp objects in the esophagus, multiple magnets or single magnet with a metallic object, or foreign bodies associated with respiratory symptoms mandate urgent removal within 12hr of presentation.

Button batteries, in particular, must be emergently removed within 2hr of presentation regardless of the timing of the patient’s last oral intake because they induce mucosal injury in as little as 1hr of contact time and involve all esophageal layers within 4hr

Asymptomatic blunut objects and coins lodged in the esophagus can be observed for up to 24hr in anticipation of passage into the stomach. “

130
Q
"Which is the most important factor in the recurrence of febrile seizures?
A. Age more than 18 months
B. Seizure lasting than 5 minutes
C. Family history of seizures
D. Long duration of fever"
A

C

“Major

  • age <1 yr
  • duration of fever <24 hr
  • fever 38-39C

Minor

  • family history of febrile seizures
  • family history of epilepsy
  • complex febrile seizure
  • daycare
  • male gender
  • lower serum Na at time of presentation

Nelson 21st p3093 Table 611.6 Risk factors for occurence of subsequent epilepsy after a febrile seizure

  • simple febrile seizure (1%)
  • recurrent febrile seizures (4%)
  • complex febrile seizures >15 min in duration or recurrent within 24hr (6%)
  • fever <1hr before febrile seizure (11%)
  • family history of epilepsy (18%)
  • complex focal febrile seizures (29%)
  • neurodevelopmental abnormalities (33%) “
131
Q
"Case of vomiting, headache and ataxia. On CT homogenous enhancing mass on the occipital area
A. Cerebellar astrocytoma
B. Ependymoma
C. Medulloblastoma
D. Brainstem glioma"
A

C

“Nelson 21st p2666
In children 0-14 yr old, the most common tumors are pilocytic astrocytomas and medulloblastoma/primitive neuroectodermal tumors (PNETS)

In adolescents (15-19 yr), the most common tumors are pituitary/craniopharyngeal tumors, and pilocytic astrocytomas.

During the first yr of life, supratentorial tumors predominate and most often include choroid plexus complex tumors and teratomas. In children 1-10 yr old, infratentorial tumors predominate because of the high incidence of juvenile pilocytic astrocytoma and medulloblastoma. After 10yr ofage, supratentorial tumors again predominate, with diffuse astrocytomas most common.

Nelson 21st p2668 Table 524.3 Posterior fossa tumors of childhood

  1. Medulloblastoma (35-40%) - heterogenously or homogenously enhancing 4th ventricular mass, may be disseminated; 65-85% survival, dependent on stage
  2. Cerebellar astrocytoma (35-40%) - cerebrellar hemisphere mass, usually with cystic and solid components, 90-100% survival in totally resected pilocytic type
  3. Brainstem glioma (10-15%) - diffusely expanded, minimally or partially enhancing mass in 80%, 20% more focal tectal or cervicomedullary lesion, >90% mortality in diffuse tumors
  4. Ependymoma (10-15%) - usually enhancing 4th ventricular mass with cerebellopontine predeliction, >75% survival if totally resected
  5. Atypical teratoid/rhabdoid (>5%, 10-15% of infantile malignant tumors) - as in medulloblastoma but more laterally extended; <20% survival”
132
Q

Management for HIE

A

Cerebral hypothermia

“Nelson 21st p920-921
Therapeutic hypothermia, whether head cooling or systemic cooling (33.5C within the first 6hr after birth and maintained for 72hr) has been shown in various trials to reduce mortality and major neurodevelopmental impairment at 18mo of age.

Infants treated with systemic hypothermia have a lower incidence of cortical neuronal injury on MRI, suggesting systemic hypothermia may result in more uniform cooling of the brain and deeper CNS structures than selective head cooling.

The therapeutic effect of hypothermia likely results from decreased neuronal injury achieved by reducing rates of apoptosis and production of neurotoxic mediators, including extracellular glutamate, free radicals, nitric oxide, and lactate. There is also benefit in seizure reduction. “

133
Q

“Cause of cerebral palsy from extrapyramidal sources
A. Asphyxia
B. Kernicterus
C. Hypoxia”

A

A

”"”Nelson 21st p3169
Athetoid CP, also called choreoathetoid, extrapyramidal, or dyskinetic CP, is less common than spastic CP and makes up approximately 15–20% of patients with CP. Affected infants are characteristically hypotonic with poor head control and marked head lag and develop variably increased tone with rigidity and dystonia over several years.

Generally, upper motor neuron signs are not present, seizures are uncommon, and intellect is preserved in many patients. This form of CP is also referred to in Europe as dyskinetic CP and is the type most likely to be associated with birth asphyxia.
“””

134
Q

“Case of cafe au lait, lisch nodules
A. Neurofibromatosis
B. Tuberous sclerosis
C. Sturgg Webber”

A

A

“Nelson 21st p3140 Neurofibromatosis
The neurofibromatoses are autosomal dominant disorders that cause tumors to grow on nerves and other systemic abnormalities.

The disease is clinically diganosed when any two of the following 7 features are present

  1. Six of more cafe au lait macules >5mm in greatest diameter in prepubertal individuals and >15mm in greatest diameter in postpubertal individuals. Cafe au lait macules are the hallmark of neurofibromatosis and are present in almost 100% of patients
  2. Axillary or inguinal freckling consisting of multiple hyperpigmented areas 2-3mm in diameter
  3. Two or more iris Lisch nodules, which are hamartomas located within the iris and are best identified by slit lamp examination
  4. Two or more neurofibromas or one plexiform neurofibroma
  5. Distinctive osseous lesion such as sphenoid dysplasia or cortical thinning of long bones with or without pseudoarthrosis
  6. Optic gliomas

Nelson 21st p3144 Table 614.4 Major features of tuberous sclerosis complex, Table 614.5 Minor features of tuberous sclerosis complex

Major

  • cortical dysplasias (including tubers and cerebral white matter migration lines)
  • subependymal nodules
  • subependymal giant cell astrocytoma
  • facial angiofibromas or forehead plaque
  • ungual fibromas
  • hypomelanotic macules
  • Shagreen patch
  • multiple retinal nodule hamartomas
  • cardiac rhabdomyoma
  • renal angiomyolipoma
  • pulmonary lymphangioleiomyomatosis

Minor

  • dental enamel pits
  • intraoral fibromas
  • retinal achromic patch
  • confetti skin lesions
  • nonrenal hamartoma
  • multiple renal cysts

*The hallmark of TSC is the involvement of the CNS. Retinal lesions consist of hamartomas (elevated mulberry lesions or plaque like lesions) and white depigmented patches (similar to hypopigmented skin lesions). The characteristic brain lesion is a cortical tuber

Nelson 21st p3146 Sturge-Weber
Sturge-Weber syndrome is a segmental vascular neurocutaneous disorder with a constellation of symptoms and signs characterized by capillary malformation on the face (port-wine birthmark) and brain (leptomeninges) as well as abnormal blood vessels of the eye leading to glaucoma.

Patients present with seizures, hemiparesis, stroke-like symptoms, headaches, and developmental delay. “

135
Q

Case of bilateral ascending paralysis motor more than sensory deficits with dysphagia and dysphonia. Diagnosis

A

Guillan Barre

136
Q

“Child a came in because of seizures. On further history was noted to have diarrhea. On physical exam Chvosteks and Trosseau was equivocal. Diagnosis?

a. Ca < 7mg/ dl
b. Na < 120me q/ L
c. K <5meq/L
d. Mg< 2”

A

B

“Nelson 21st p397
Brain cell swelling is responsible for most of the symptoms of hyponatremia. Neruologic symptoms of hyponatremia include anorexia, nausea, emesis, malaise, lethargy, confusion, agitation, headache, seizures, coma, and decreased reflexes. Patients may have hypothermia and Cheyne-Stoke respiration. Acute sever hyponatremia can cause brainstem herneation and apnea, respiratory support is often necessary. “

137
Q

“3-month-old infant came in due to generalized seizure for the first time with fever. No other complications . Mother is worried about recurrence. What will you advise?

a. The younger the age at onset, the higher risk to develop epilepsy in the future
b. Repeated simple focal seizures will not cause permanent brain damage
c. It will lead to brain damage
d. Need to work up for metabolic problem”

A

B

“Nelson 21st p3092-3093
Febrile seizures are seizures that occur between the ages of 6 and 60mo (peak 12-18mo) with a temperature of 38C or higher, that are not the result of CNS infection or metabolic imbalance, and that occur in the absence of a history of prior afebrile seizures.

Between 2% and 5% of neurologically health infants and children experience at least one, usually simple, febrile seizure. Simple febrile seizures do not have an increased risk of mortality even though they are concerning to patients.

There are no long-term adverse effects of having one or more simple febrile seizures. Although approximately 15% of children with epilepsy have had febrile seizures, only 5% of children who experience febrile seizures proceed to develop epilepsy. “

138
Q
"Patient with febrile seizure. Risk factor for development of epilepsy?
A. Simple febrile seizure
B. Focal complex febrile seizure
C. Neurodevelopmental abnormalities
D. Family history of epilepsy"
A

C

“Nelson 21st p3093. Table 611.6 Risk factors for occurence of subsequent epilepsy after a febrile seizure.

Simple febrile seizure 1%
Recurrent febrile seizure 4%
Complex febrile seizures (>15 min in duration or recurrent within 24hr) 6%
Fever <1hr before febrile seizure 11%
Family history of epilepsy 18% 
Complex febrile seizures (focal) 29%
Neurodevelopmental anomalies 33%"
139
Q

“Facial weakness in a child with poor head control and with drooling only able to swallow 10 ml of milk during feeding, difficult to swallow solids. What kind of CP?

a. Spastic hemiplegia
b. Spastic diplegia
c. Spastic quadriplegia
d. Extrapyramidal”

A

C

“Nelson 21st p3168-3169
CP is generally divided on several major motor syndrome that differ according to the pattern of neurologic involvement, neuropathology, and etiology.

Infants with spastic hemiplegia have decreased spontaneous movement on the affected side and show hand preference at a very early age. The arm is often more involved than the leg and difficulty in hand manipulation is obvious by 1 yr of age. Walking is usually delayed until 18-24mo, and a circumductive gait is apparent.

Spastic diplegia is a bilateral spasticity of the legs that is greater than the arms. The first clinical manifestation of spastic diplegia is often noted when the affected infant begins to crawl. The child uses the arms in normal reciprocal fashion but tends to drag the legs behind more as a ruddur (commando crawl)

Spastic quadriplegia is the most severe form of CP because of marked motor impairment of all extremities and the high association with intellectual disability and seizures. Swallowing difficulties are common as a result of supranuclear bulbar palsies, often leading to aspiration pneumonia and growth failure. Neurologic examination shows increased tone and spasticity of all extremities, decreased spontaneous movements, brisk reflexes, and plantar extensor responses. Associated developmental disabilities, including speech and visual anomalies, are particularly prevalent in this group of children.

Athetoid CP, also called choreoathetoid, extrapyramidal, or dyskinetic CP, is less common (15-20%). Affected infants are characteristically hypotonic with poor head control and marked head lag and develop variably increased tone with rigidity and dystonia over several years. The term dystonia refers to the abnormality in tone in which muscles are rigid throughout their range of motion and involuntary contractions can occur in both flexors and extensors leading to limb positioning in fixed postures.

Nelson 21st p3169 Table 616.1 Classification of cerebral palsy and major causes

  1. Spastic diplegia (35%) - prematurity, ischemia, infection, endocrine/metabolic (e.g. thyroid)
  2. Spastic quadriplegia (20%) - ischemia, infection, endocrine/metabolic, genetic/developmental
  3. Hemiplegia (25%) - thrombophilic disorders, infection, genetic/developmental, periventricular hemorrhagic infarction
  4. Extrapyramidal (athetoid/dyskinetic) (15%) - asphyxia, kernicterus, mitochondrial, genetic/metabolic”
140
Q

“A patient complaining of headache and fever, previously given amoxicillin for respiratory tract infection for three days. But had neck pain and rigidity. He was admitted and had CSF findings of increased opening pressure, WBC 5000, predominantly monocytes, glucose 40mg/dl, protein 150mg/d
A. TB meningitis
B. Acute bacterial meningitis
C. Partially treated bacterial meningitis
D. Viral encephalitis”

A

C

“Nelson 21st p3222 Table 621.1 CSF fluid findings in CNS disorders

Normal

  • pressure <28
  • WBC: <5, >=75% lymphocytes; in neonates <20
  • protein: 20-45
  • glucose >50 or 75% serum glucose

Acute bacterial meningitis

  • elevated pressure
  • WBC: 100-10,000, PMN predominance
  • protein: 100-500
  • glucose: decreased, usually <40 or <50% of serum glucose

Partially treated bacterial meningitis

  • normal or elevated pressure
  • WBC 5-10,000, PMN usual but mononuclear cells may predominate if with extended pretreatment
  • protein: 100-500
  • glucose: normal or decreased

Viral meningitis

  • normal or slightly elevated pressure
  • WBC rarely >1000, PMN early, but mononuclear cells predominate through most of the course
  • protein: 50-200
  • glucose: generally normal, may be decreased <40 in some viral diseases, particularly mumps

TB meningitis

  • usually elevated pressure
  • WBC 10-500, PMN early, but lymphocytes predominate through most of the course
  • protein: 100-3000, may be higher in presence of block
  • glucose <50 in most cases, decreases with time if treatment is not provided

Fungal meningitis

  • usually elevated pressure
  • WBC 5-500, PMn early but mononuclear cells predominate for most of the course
  • cryptococcal meningitis may lack pleiocytosis, coccidioidal meningitis may have eosinophilia
  • protein: 25-500
  • glucose: <50, decreases with time if treatment is not provided

Syphilis and leptospirosis

  • usually elevated pressure
  • WBC 50-500, lymphocytes predominate
  • protein: 50-200
  • glucose usually normal “
141
Q

“Patient with weakness on lower extremities, ascending pattern. CSF with WBC 5, normal glucose, elevated protein
A. Transverse myelitis
B. Guillain -Barre syndrome
C. Polio”

A

B

“Nelson 21st p3337
CSF studies are helpful in diagnosing GBS. The CSF protein is usually elevated to more than twice the upper limit of normal, the glucose level is normal, and there is no pleocytosis; there should be fewer than 10 white blood cells/mm3. Bacterial cultures are negative, whereas viral studies rarely isolate specific viruses. The dissociation between high CSF protein and a lack of cellular response (cytoalbuminologic dissociation) in a patient with an acute or subacute polyneuropathy is essentially diagnostic of GBS. These findings may not be apparent in the first week after the onset of symptoms”

142
Q

“A 8 year old girl increase day dreaming, blank stares . EEG shows 3 sec spike and generalized wave

a. Absence Seizure
b. Generalized Tonic clonic seizure
c. Juvenile myoclonic epilepsy
d. None of the above”

A

A

“Nelson 21st p3095
Absence seizures are generalized seizures consisting of staring, unresponsiveness, and eye flutter lasting usually for a few seconds.

Typical absences are associated with 3Hz spike-and-slow-wave discharges and with childhood absence epilepsy which has a good prognosis.

Atypical absences are associated with 1- to 2-Hz spike-and-slow-wave discharges, and with head atonia and myoclonus during the seizures. They occur in Lennox-Gastaut syndrome and similar syndromes, which have a poor prognosis. “

143
Q

“Which of the following features is NOT characteristic of complex febrile seizures?
A. Repeated convulsions occur within the same day.
B. The seizures last longer than 5 minutes.
C. There is focal seizure activity.
D. There are focal findings during the post-ictal period.”

A

B

“Nelson 21st p3092
A simple febrile seizure is a primary generalized, usually tonic-clonic attack associated with fever, lasting for a maximum of 15min, and not recurrent within a 24hr period.

A complex febrile seizure is more prolonged (>15min), and/or is focal, and/or recurs within 24hrs.

Febrile status epilepticus is a febrile seizure lasting longer than 30 min. Most patients with simple febrile seizures have a very short postictal state and usually return to their baseline normal behavior and consciousness within minutes of the seizure. “

144
Q

“A 2-year old male consulted because of fever, vomiting, increased sleeping time & convulsions . On examination, he had
nuchal rigidity & cranial nerve palsies . The CSF analysis & cranial scan were suggestive of TB meningitis. In what stage of the disease is he?
A. 1
B. 2
C. 3
D. 4”

A

B

“TBIC 2016 p52
The first stage is characerized by personality changes, irritability, anorexia, listlessness, and some fever. After 1 or 2 weeks, the second stage begins with signs of increased intracranial pressure and cerebral damage appearing: drowsiness, stiff neck, cranial nerve palsies, inequality of the pupils, vomiting, tache cerebrale, absence of abdominal reflexes, and convulsions that may be tonic or clonic, focal or generalized. The third stage is characterized by coma, irregular pulse and respirations, and rising fever. “

145
Q

“What increases the risk for NEC?

a. Minimal enteral feeding
b. Drugs that inhibit gastric acid secretion
c. Prophylactic enteral antibiotics
d. Probiotics”

A

B

“Nelson 21st p953
Other preventive strategies using prebiotics and synbiotics have also been studied, with variable outcomes. Inhibitors of gastric acid secretion (H2-receptor blockers, proton pump inhibitors) or prolonged empirical antibiotics in the early neonatal period have been associated with increased risk of NEC and should be avoided.

Nelson 21st p951
3 major risk factors have been implicated: prematurity, bacterial colonization of the gut, and formula feeding. NEC develops primarily in premature infants with exposure to metabolic substrate in the context of immature intestinal immunity, microbial dysbiosis, and mucosal ischemia

The greatest risk factor for NEC is prematurity. NEC rarely occurs before the initiation of enteral feeding and is much less common in infants fed human milk. Aggressive enteral feeding may predispose to the development of NEC.”

146
Q

“Most common cause of antenatal fetal distress?

a. OM
b. Multiple pregnancy
c. Hypertension
d. Uteroplacental insufficiency”

A

D

“Nelson 21st p881
The predominant cause of antepartum fetal distress is uteroplacental insufficiency, which may manifest clinically as IUGR, fetal hypoxia, increased vascular resisteance iin fetal blood vessels, and, when severe, mixed respiratory and metabolic acidosis. “

147
Q

“Blood transfusion not indicated?

a. Asymptomatic preterm with Hgb 7 g/dl
b. Asymptomatic term infant with Hgb 10 g/dl
c. Infants with HMD on respiratory support
d. Symptomatic term infant with Hgb 8 g/dl”

A

B

“Nelson 21st p966 Table 124.4 Suggested transfusion thresholds (preterm infants)

Presence of respiratory support
Week 1: Hgb 11.5, Hct 35%
Week 2: Hgb 10.0, Hct 30%
Week 3: Hgb 8.5, Hct 25%

Absence of respiratory support
Week 1: Hgb 10.0, Hct 30%
Week 2: Hgb 8.5, Hct 25%
Week 3: Hgb 7.5, Hct 23%

  • In addition to these factors, transfusion should be considered for infants with acute blood loss >20% or significant hemolysis, as well as before surgery
  • Transfusion should be based on hemodynamic stability, respiratory status, overall clinical condition, and laboratory values

Nelson 21st p2581 Table 497.1 Guidelines for pediatric blood cell transfusions

Infants <=4mo old

  1. Maintain hgb >12 g/dl in severe pulmonary disease
  2. Maintain hgb >12 g/dl during ECMO
  3. Maintain hgb >10 g/dl in moderate pulmonary disease
  4. Maintain hgb >12 g/dl in severe cardiac disease
  5. Maintain hgb >10 g/dl postoperatively and during major surgery
  6. Maintain hgb >7 g/dl postoperatively
  7. Maintain hgb >7 g/dl in symptomatic anemia “
148
Q

“Excessively small anterior & posterior fontanel can be found in?

a. Congenital rubella syndrome
b. IUGR
c. Congenital hyperthyroidism”

A

C

“Nelson 21st p869
Persistently small fontanels suggest microcephaly, craniosynostosis, congenital hyperthyroidism, or wormian bones; presence of a 3rd fontanel suggests trisomy 21 but is seen in preterm infants.

Nelson 21st p869 Table 113.1 Disorders associated with a large anterior fontanel

  • Hypothyroidism
  • Achondroplasia
  • Apert syndrome
  • Cleidocranial dysostosis
  • Congenital rubella syndrome
  • Hallerman-Streiff syndrome
  • Hydrocephaly
  • Hypophosphatasia
  • Intrauterine growth restriction
  • Kenny syndrome
  • Osteogenesis imperfecta
  • Prematurity
  • Pyknodysostosis
  • Russel-Silver syndrome
  • Trisomy 13, 18, 21
  • Vitamin D deficiency rickets “
149
Q

“Mother is on heroin while pregnant. True of effects of heroin on newborn EXCEPT

a. Increase risk of congenital anomaly
b. Decrease incidence of RDS
c. Decrease hyperbilirubinemia”

A

A

150
Q
"Domain of intelligence that enables child to coordinate motor and sensory skills
A. Language
B. Adaptive
C. Psychosocial
D. Gross motor"
A

D